Strat OG SBA 2018 Obstetrics
Strat OG SBA 2018 Obstetrics
(/)
Obstetrics page 1
Assessment Total A empts: 4 Highest Score: 100 %
History
View last results (/quiz/latest/305364/299342) View highest
result (/quiz/highest/305364/299342) Retake quiz
(/quiz/retake/299342)
A 40-year-old woman is seen in the antenatal clinic at 20 weeks of gesta on. Both
Q her booking and anomaly scan are normal. She has a BMI of 24. She had a
previous vaginal delivery at 39 weeks of gesta on of a baby weighing 1.8 kg. She
smokes 20 cigare es per day. What is the next most appropriate inves ga on?
https://stratog.rcog.org.uk/sbas-part-2-mrcog-online-resource/obstetric-sbas/obstetrics-page-1 1/6
11/21/2018 Obstetrics page 1 | StratOG
Isoenzyme MB
LDH (lactate dehydrogenase)
Myoglobin
Troponin I
The obstetric team are conduc ng a study to evaluate whether there has been any
Q effect on pa ent sa sfac on following the establishment of an outpa ent
induc on of labour (IOL) programme. Women undergoing inpa ent IOL and
women undergoing outpa ent IOL were asked to rate their overall sa sfac on
with the process using a visual analogue scale from 1 (least sa sfied) to 10 (most
sa sfied). What is the most appropriate sta s cal test to assess whether there is a
significant difference in sa sfac on between the two groups?
Student’s t test
Wilcoxon matched pairs signed rank test
A woman a ends the antenatal clinic at 30 weeks of gesta on and discloses that
Q she had suspected whooping cough 2 months earlier. What is the single best
recommenda on regarding pertussis immunisa on?
https://stratog.rcog.org.uk/sbas-part-2-mrcog-online-resource/obstetric-sbas/obstetrics-page-1 3/6
11/21/2018 Obstetrics page 1 | StratOG
You are asked to repair a vaginal tear following a normal delivery. The mother’s
Q weight is 60 kg. She is otherwise well with no allergies. What is the maximum dose
of lidocaine 1% without epinephrine that you can use for perineal infiltra on?
8 ml (80 mg)
12 ml (120 mg)
18 ml (180 mg)
24 ml (240 mg)
36 ml (360 mg)
A woman who is 24 weeks pregnant contacts the maternity day unit repor ng
Q possible exposure to facial shingles 4 days earlier. The pregnant woman believes
she has had chickenpox when she was a child. What advice should she be given?
Offer tes ng for varicella zoster virus (VZV) immunity and, if non-immune,
offer varicella zoster immunoglobulin (VZIG)
https://stratog.rcog.org.uk/sbas-part-2-mrcog-online-resource/obstetric-sbas/obstetrics-page-1 4/6
11/21/2018 Obstetrics page 1 | StratOG
Tell her to report the development of a rash, and if it develops, offer her
treatment with oral aciclovir
Tell her to report the development of a rash and, if it develops, offer her
treatment with VZIG
A pregnant woman with a BMI of 25 sees her midwife at 24 weeks of gesta on. A
Q single symphysis fundal height (SFH) measurement is undertaken which is less
than expected for this gesta on. What is the most appropriate management?
https://stratog.rcog.org.uk/sbas-part-2-mrcog-online-resource/obstetric-sbas/obstetrics-page-1 5/6
11/21/2018 Obstetrics page 1 | StratOG
Pa ent Associa on
This is the last ques on of this quiz. Press finish to submit your choice(s) and reveal the answer(s).
https://stratog.rcog.org.uk/sbas-part-2-mrcog-online-resource/obstetric-sbas/obstetrics-page-1 6/6
11/21/2018 Obstetrics page 1 | StratOG
(/)
Obstetrics page 1
Assessment Total A empts: 4 Highest Score: 100 %
History
View last results (/quiz/latest/305364/299342) View highest
result (/quiz/highest/305364/299342) Retake quiz
(/quiz/retake/299342)
A Your answer:
> Umbilical artery Doppler at 26–28 weeks of gesta on
Correct answer:
> Umbilical artery Doppler at 26–28 weeks of gesta on
https://stratog.rcog.org.uk/sbas-part-2-mrcog-online-resource/obstetric-sbas/obstetrics-page-1 1/7
11/21/2018 Obstetrics page 1 | StratOG
A Your answer:
> Late indirect maternal death
Correct answer:
> Late indirect maternal death
The correct answer is late indirect maternal death. A maternal death
that occurs 6 weeks following child birth is termed as late maternal
death. If death occurs of a pre-exis ng medical condi on it is called
an indirect maternal death. See Maternal, Newborn and Infant
Clinical Outcome Review Programme. Saving Lives, Improving
Mother's Care. Lessons learned to inform future maternity care from
the UK and Ireland Confiden al Enquiries into Maternal Deaths and
Morbidity 2009–2012. Oxford: Na onal Perinatal Epidemiology Unit,
University of Oxford. 2014. (h p://www.npeu.ox.ac.uk/mbrrace-
uk/reports)
https://stratog.rcog.org.uk/sbas-part-2-mrcog-online-resource/obstetric-sbas/obstetrics-page-1 2/7
11/21/2018 Obstetrics page 1 | StratOG
A Your answer:
> 0.5 mg (0.5 ml of 1:1000)
Correct answer:
> 0.5 mg (0.5 ml of 1:1000)
The correct answer is 0.5 mg (0.5 ml of 1:1000). The correct dose of
intramuscular (im) adrenaline in anaphylac c shock is 0.5mg. Doses
of 0.01 mg, 0.05mg and 0.1 mg are too small for therapeu c effect
in circulatory collapse by im route and would be more appropriate
doses for iv route. 10mg is too large for an ini al dose but if there is
a subop mal response to ini al dose, then injec ons should be
repeated every 10 minutes and may therefore reach an
accumula ve dose of 10 mg. See the Bri sh Na onal Formulary
(h p://www.bnf.org/bnf/index.htm).
A Your answer:
> Troponin I
Correct answer:
> Troponin I
The correct asnwer is Troponin I. Troponin I is unaffected by labour,
anaesthesia or delivery. See Wuntakal R, She y N, Ioannou E,
Sharma S, Kurian J. Myocardial infarc on and pregnancy. The
Obstetrician & Gynaecologist 2013;15:247–55.
(h p://onlinelibrary.wiley.com/doi/10.1111/tog.12052/full)
https://stratog.rcog.org.uk/sbas-part-2-mrcog-online-resource/obstetric-sbas/obstetrics-page-1 3/7
11/21/2018 Obstetrics page 1 | StratOG
The obstetric team are conduc ng a study to evaluate whether there has
Q been any effect on pa ent sa sfac on following the establishment of an
outpa ent induc on of labour (IOL) programme. Women undergoing
inpa ent IOL and women undergoing outpa ent IOL were asked to rate
their overall sa sfac on with the process using a visual analogue scale from
1 (least sa sfied) to 10 (most sa sfied). What is the most appropriate
sta s cal test to assess whether there is a significant difference in
sa sfac on between the two groups?
A Your answer:
> Mann Whitney U test
Correct answer:
> Mann Whitney U test
The correct answer is the Mann Whitney U test. See Campbell MJ,
Machin D, Walters SJ. Medical sta s cs: a textbook for the health
sciences (medical sta s cs). Wiley-Blackwell. 2007.
A Your answer:
> Maternal vaccina on should be given now
Correct answer:
> Maternal vaccina on should be given now
The correct answer is that maternal vaccina on should be given
now. Despite high vaccina on coverage in Britain since the 1990s,
pertussis con nues to display 3–4 yearly peaks in ac vity. In 2012
there was a major leap in pertussis, with levels above those
reported in the previous 20 years. It was seen in all age groups.
Infants under 3 months are at highest risk of complica ons and
death. In view of the outbreak in 2012 all pregnant women are
offered pertussis vaccina on during pregnancy.
https://stratog.rcog.org.uk/sbas-part-2-mrcog-online-resource/obstetric-sbas/obstetrics-page-1 4/7
11/21/2018 Obstetrics page 1 | StratOG
You are asked to repair a vaginal tear following a normal delivery. The
Q mother’s weight is 60 kg. She is otherwise well with no allergies. What is
the maximum dose of lidocaine 1% without epinephrine that you can use
for perineal infiltra on?
A Your answer:
> 18 ml (180 mg)
Correct answer:
> 18 ml (180 mg)
The correct answer is 18 ml (180 mg). The maximum dose of
lidocaine is 3 mg/kg. As the woman's weight is 60 kg, the dose is 3 x
60 = 180 mg total dose. 1% lidocaine contains 1 x 10 mg/ml = 10
mg/ml. Therefore the maximum volume is 180 /10 = 18 ml of 1%
lidocaine. See StratOG Core Training eTutorial on Obstetric analgesia
and anaesthesia (h ps://stratog-live.rcog.org.uk/tutorials/core-
training/management-labour-and-delivery/obstetric-analgesia-and-
anaesthesia) and Anaesthesia UK. Pharmacology of regional
anaesthesia. Accessed online 27 January 2015
(h p://www.frca.co.uk/ar cle.aspx?ar cleid=100816).
A Your answer:
> Offer tes ng for varicella zoster virus (VZV) immunity and, if non-
immune, offer varicella zoster immunoglobulin (VZIG)
Correct answer:
> Offer tes ng for varicella zoster virus (VZV) immunity and, if non-
immune, offer varicella zoster immunoglobulin (VZIG)
The correct answer is offer tes ng for varicella zoster virus (VZV)
immunity and, if non-immune, offer varicella zoster immunoglobulin
(VZIG). VZV is highly contagious and can be transmi ed by
respiratory droplets, direct personal contacts or fomites. It is
possible to catch it from both chickenpox and herpes zoster (HZ) but
it is highly unlikely if the HZ is in non-exposed sites. VZIG is effec ve
https://stratog.rcog.org.uk/sbas-part-2-mrcog-online-resource/obstetric-sbas/obstetrics-page-1 5/7
11/21/2018 Obstetrics page 1 | StratOG
A Your answer:
> Refer if SFH measurement on a customised chart plots below the
10th cen le
Correct answer:
> Refer if SFH measurement on a customised chart plots below the
10th cen le
The correct answer is refer if SFH measurement on a customised
chart plots below the 10th cen le. Abdominal palpa on is poor at
predic ng small-for-gesta onal-age (SGA) babies, especially in a
mixed risk popula on. SFH using a customised growth chart which
takes into account maternal height, weight, parity and ethnic group
improves the predic on of SGA babies, but there is wide varia on in
the predic ve accuracy ranging from a sensi vity of 27–86% and a
specificity of 80–93%. See Royal College of Obstetricians and
Gynaecologists. The inves ga on and management of the small–
for–gesta onal–age fetus. Green-top guideline 31. London: RCOG.
2014. (h ps://www.rcog.org.uk/en/guidelines-research-
services/guidelines/gtg31/)
https://stratog.rcog.org.uk/sbas-part-2-mrcog-online-resource/obstetric-sbas/obstetrics-page-1 6/7
11/21/2018 Obstetrics page 1 | StratOG
A Your answer:
> Pa ent Advice and Liaison Service
Correct answer:
> Pa ent Advice and Liaison Service
The correct answer is the Pa ent Advice and Liaison Service. If a
pa ent has a complaint or concern, it is best dealt with by the
provider of the health care in the first instance. Other organisa ons
may be appropriate if the ini al response is not sa sfactory.
https://stratog.rcog.org.uk/sbas-part-2-mrcog-online-resource/obstetric-sbas/obstetrics-page-1 7/7
11/21/2018 Obstetrics page 2 | StratOG
(/)
Obstetrics page 2
Assessment Total A empts: 2 Highest Score: 100 %
History
View last results (/quiz/latest/305488/299343) View highest
result (/quiz/highest/305488/299343) Retake quiz
(/quiz/retake/299343)
Cor sol
Estrogen
https://stratog.rcog.org.uk/sbas-part-2-mrcog-online-resource/obstetric-sbas/obstetrics-page-2 1/6
11/21/2018 Obstetrics page 2 | StratOG
0%
25%
50%
75%
100%
https://stratog.rcog.org.uk/sbas-part-2-mrcog-online-resource/obstetric-sbas/obstetrics-page-2 2/6
11/21/2018 Obstetrics page 2 | StratOG
Emergency caesarean
Postpartum haemorrhage
S llbirth
Venous thromboembolism
A pregnant woman is iden fied as being suscep ble to rubella from her first
Q trimester booking blood results. When discussing this result at the next antenatal
clinic appointment, what is the most appropriate advice that she should be given?
https://stratog.rcog.org.uk/sbas-part-2-mrcog-online-resource/obstetric-sbas/obstetrics-page-2 3/6
11/21/2018 Obstetrics page 2 | StratOG
Dexamethasone
S-adenosyl methionine
Topical emollients
Ursodeoxycholic acid
Vitamin K
Abnormal CTG
https://stratog.rcog.org.uk/sbas-part-2-mrcog-online-resource/obstetric-sbas/obstetrics-page-2 4/6
11/21/2018 Obstetrics page 2 | StratOG
1 year
2 years
3 years
4 years
5 years
A 36-year-old woman a ends the antenatal clinic at 20 weeks of gesta on. She has
Q had three previous caesarean sec ons and has a normal placental site. She
consented for another caesarean sec on. What is the most likely surgical
complica on?
Bladder injury
Blood transfusion
Bowel injury
https://stratog.rcog.org.uk/sbas-part-2-mrcog-online-resource/obstetric-sbas/obstetrics-page-2 5/6
11/21/2018 Obstetrics page 2 | StratOG
Fetal lacera on
Hysterectomy
This is the last ques on of this quiz. Press finish to submit your choice(s) and reveal the answer(s).
https://stratog.rcog.org.uk/sbas-part-2-mrcog-online-resource/obstetric-sbas/obstetrics-page-2 6/6
11/21/2018 Obstetrics page 2 | StratOG
(/)
Obstetrics page 2
Assessment Total A empts: 2 Highest Score: 100 %
History
View last results (/quiz/latest/305488/299343) View highest
result (/quiz/highest/305488/299343) Retake quiz
(/quiz/retake/299343)
A Your answer:
> Human placental lactogen
Correct answer:
> Human placental lactogen
The corerct answer is human placental lactogen. See Nelson-Piercy
C. Handbook of obstetric medicine. Fourth edi on. CRC Press. 2010.
https://stratog.rcog.org.uk/sbas-part-2-mrcog-online-resource/obstetric-sbas/obstetrics-page-2 1/7
11/21/2018 Obstetrics page 2 | StratOG
A Your answer:
> 50%
Correct answer:
> 50%
The correct answer is 50%. Haemophilia A is an X-linked recessive
disorder so 50% of her sons will be affected and 50% of her
daughters will be carriers.
A Your answer:
> Prescribe intrapartum an bio c prophylaxis with any
dura on of prelabour rupture of membranes
Correct answer:
> Prescribe intrapartum an bio c prophylaxis with any
dura on of prelabour rupture of membranes
The correct answer is prescribe intrapartum an bio c prophylaxis
with any dura on of prelabour rupture of membranes. In preterm
labour, an bio cs should be considered if membranes rupture at
any me prior to the onset of labour. If the woman is at term,
an bio cs should only be given if the woman has had a posi ve
culture for GBS in this pregnancy or has clinical signs of infec on.
See Na onal Ins tute for Health and Clinical Excellence. Neontal
infec on (early onset): an bio cs for preven on and treatment.
CG149. London: NICE; 2012.
(h p://www.nice.org.uk/guidance/cg149)
https://stratog.rcog.org.uk/sbas-part-2-mrcog-online-resource/obstetric-sbas/obstetrics-page-2 2/7
11/21/2018 Obstetrics page 2 | StratOG
A Your answer:
> Venous thromboembolism
Correct answer:
> Venous thromboembolism
The correct answer is venous thromboembolism. The risk of
diabetes is about three mes higher. The risk of hypertensive
disease is two-to-three mes higher. Caesarean sec on, s llbirth
and postpartum haemorrhage are about twice as likely in women
with a high BMI. Venous thromboembolism is, however, nine mes
higher in this group. See the CMACE/RCOG Joint Guideline.
Management of women with obesity in pregnancy. CMACE. 2010
(h ps://www.rcog.org.uk/globalassets/documents/guidelines/cmac
ercogjointguidelinemanagementwomenobesitypregnancya.pdf).
A pregnant woman is iden fied as being suscep ble to rubella from her
Q first trimester booking blood results. When discussing this result at the next
antenatal clinic appointment, what is the most appropriate advice that she
should be given?
A Your answer:
> A single dose of MMR should be offered immediately postnatally
with a second dose at the six-week postnatal check
Correct answer:
> A single dose of MMR should be offered immediately postnatally
with a second dose at the six-week postnatal check
https://stratog.rcog.org.uk/sbas-part-2-mrcog-online-resource/obstetric-sbas/obstetrics-page-2 3/7
11/21/2018 Obstetrics page 2 | StratOG
A Your answer:
> Ursodeoxycholic acid
Correct answer:
> Ursodeoxycholic acid
The correct answer is ursodeoxycholic acid. Pruri s in pregnancy is
common, affec ng nearly a quarter of pregnant women. Obstetric
cholestasis is diagnosed when abnormal liver func on tests are
found in associa on with pruri s. Normal pregnancy values should
be used with an upper limit of normal 20% below nonpregnant
levels for transaminases, γ-glutamyl transferase and bilirubin.
Alkaline phosphatase is generally raised in pregnancy due to
placental produc on.
Topical emollients may provide temporary relief of pruri s. S-
adenosyl methionine is not recommended and dexamethasone
should only be used as part of a trial. Vitamin K should be
prescribed if the prothrombin me is prolonged, but is not an
effec ve treatment. See Royal College of Obstetricians and
https://stratog.rcog.org.uk/sbas-part-2-mrcog-online-resource/obstetric-sbas/obstetrics-page-2 4/7
11/21/2018 Obstetrics page 2 | StratOG
A Your answer:
> Abnormal CTG
Correct answer:
> Abnormal CTG
The correct answer is abnormal CTG. Vaginal birth a er an
uncomplicated lower segment caesarean sec on is successful in 72–
76% of women. The risk of uterine rupture is 22–74/10 000 (0.22–
0.74%). This is lower if the woman labours preterm (34/10 000 vs
74/10 000). An abnormal CTG is the most consistent finding in
dehiscence, occurring in 55–87% of cases. See Royal College of
Obstetricians and Gynaecologists. Birth a er previous caesarean
birth. Green-top Guideline 45. London: RCOG; 2007.
(h ps://www.rcog.org.uk/en/guidelines-research-
services/guidelines/gtg45/)
A Your answer:
> Coronary artery dissec on
Correct answer:
> Coronary artery dissec on
https://stratog.rcog.org.uk/sbas-part-2-mrcog-online-resource/obstetric-sbas/obstetrics-page-2 5/7
11/21/2018 Obstetrics page 2 | StratOG
A Your answer:
> 1 year
Correct answer:
> 1 year
The correct answer is 1 year. The majority of bariatric surgery is
carried out on women of childbearing years. Current advice is to
delay concep on for a year. However, data to support this
recommenda on is lacking, with many studies showing no
difference in outcomes in those women conceiving earlier than 12
months and those conceiving later. See Khan R , Dawlatly B,
Chappa e O. Pregnancy outcome following bariatric surgery. The
Obstetrician & Gynaecologist 2013;15:37–43
(h p://onlinelibrary.wiley.com/doi/10.1111/j.1744-
4667.2012.00142.x/full).
A Your answer:
> Blood transfusion
Correct answer:
> Blood transfusion
The correct answer is blood transfusion. Elec ve repeat caesarean
sec on is associated with increasing risks that rise with each
successive pregnancy. Blood transfusion rises from 7.9% with a third
caesarean sec on to 14.1% with the fi h caesarean. See Royal
College of Obstetricians and Gynaecologists. Birth a er previous
caesarean birth. Green-top Guideline 45. London: RCOG; 2007
(h ps://www.rcog.org.uk/en/guidelines-research-
services/guidelines/gtg45/).
https://stratog.rcog.org.uk/sbas-part-2-mrcog-online-resource/obstetric-sbas/obstetrics-page-2 7/7
11/21/2018 Obstetrics page 3 | StratOG
(/)
Obstetrics page 3
Assessment Total A empts: 2 Highest Score: 90 %
History
View last results (/quiz/latest/305390/299344) View highest
result (/quiz/highest/305390/299344) Retake quiz
(/quiz/retake/299344)
You are asked to review a woman following a forceps delivery. She presents with
Q le lateral calf paraesthesia, sensory loss between her first and second toes and
foot drop with inversion. Which nerve compression is the likely cause of her
symptoms?
https://stratog.rcog.org.uk/sbas-part-2-mrcog-online-resource/obstetric-sbas/obstetrics-page-3 1/6
11/21/2018 Obstetrics page 3 | StratOG
The midwives on the postnatal ward are concerned about the behaviour of a first
Q me mother, who they are about to discharge home. They ask you to review her.
She had an elec ve caesarean sec on for a breech presenta on 3 days ago. She is
otherwise fit and well, but has a past history of depression. Which symptoms
would concern you the most and lead you to the diagnosis of postpartum
psychosis?
A 25-year-old pregnant woman with sickle cell disease a ends the antenatal clinic
Q at 8 weeks of gesta on. What prenatal tes ng should be discussed in the first
instance?
Amniocentesis
https://stratog.rcog.org.uk/sbas-part-2-mrcog-online-resource/obstetric-sbas/obstetrics-page-3 2/6
11/21/2018 Obstetrics page 3 | StratOG
A primigravida wishes to opt for epidural analgesia in labour at term but she has
Q heard that regional analgesia increases the risk of opera ve vaginal delivery which
she is keen to avoid. Assuming she opts for an epidural analgesia, how can the
second stage of labour be managed to reduce this risk for her?
Ampicillin orally
Cefalexin orally
Cefuroxime intravenously
diagnosis of malaria?
Blood culture
Postural management
https://stratog.rcog.org.uk/sbas-part-2-mrcog-online-resource/obstetric-sbas/obstetrics-page-3 4/6
11/21/2018 Obstetrics page 3 | StratOG
A woman presents for booking in the first trimester, she is taking lithium for her
Q mental health. How o en should her serum lithium levels be checked?
A 25-year-old woman is found to have a platelet count of 110 x 10*9/l when tested
Q rou nely at 28 weeks of gesta on. Her platelet count at 12 weeks of gesta on was
352 x 10*9/l. She has no history of illness. What is the most likely diagnosis from
the list below?
HIV
https://stratog.rcog.org.uk/sbas-part-2-mrcog-online-resource/obstetric-sbas/obstetrics-page-3 5/6
11/21/2018 Obstetrics page 3 | StratOG
Immune thrombocytopenia
Thrombocytosis
This is the last ques on of this quiz. Press finish to submit your choice(s) and reveal the answer(s).
https://stratog.rcog.org.uk/sbas-part-2-mrcog-online-resource/obstetric-sbas/obstetrics-page-3 6/6
12/8/2018 Obstetrics page 3 | StratOG
(/)
You have 2 tutorial(s) due for renewal, visit your account page (/user) to renew.
Obstetrics page 3
Assessment Total A empts: 3 Highest Score: 100 %
History
View last results (/quiz/latest/305390/299344) View highest
result (/quiz/highest/305390/299344) Retake quiz
(/quiz/retake/299344)
You are asked to review a woman following a forceps delivery. She presents
Q with le lateral calf paraesthesia, sensory loss between her first and second
toes and foot drop with inversion. Which nerve compression is the likely
cause of her symptoms?
A Your answer:
> Common peroneal nerve
Correct answer:
> Common peroneal nerve
https://stratog.rcog.org.uk/sbas-part-2-mrcog-online-resource/obstetric-sbas/obstetrics-page-3 1/8
12/8/2018 Obstetrics page 3 | StratOG
The midwives on the postnatal ward are concerned about the behaviour of
Q a first me mother, who they are about to discharge home. They ask you to
review her. She had an elec ve caesarean sec on for a breech presenta on
3 days ago. She is otherwise fit and well, but has a past history of
depression. Which symptoms would concern you the most and lead you to
the diagnosis of postpartum psychosis?
A Your answer:
> Bewilderment and perplexity
Correct answer:
> Bewilderment and perplexity
The correct answer is bewilderment and perplexity. Most of these
symptoms are features of 'baby blues' which affects 30–80% of
births in the first week postpartum. Confusion, bewilderment and
perplexity are worrying symptoms and should alert you to the
diagnosis of postpartum psychosis. See Di Florio A, Smith S, Jones I.
Postpartum psychosis. The Obstetrician & Gynaecologist
2013;15:145–50
(h p://onlinelibrary.wiley.com/doi/10.1111/tog.12041/full).
A Your answer:
> Partner tes ng
https://stratog.rcog.org.uk/sbas-part-2-mrcog-online-resource/obstetric-sbas/obstetrics-page-3 2/8
12/8/2018 Obstetrics page 3 | StratOG
Correct answer:
> Partner tes ng
The correct answer is partner tes ng. Ideally this will have been
ascertained this in advance. Preconcep on counselling is very
important if the couple are iden fied as an 'at risk couple'. This is
not just if her partner carries HbS, but also if there are other
condi ons detected, e.g. β-thalassaemia or HbC.
A Your answer:
> Allow up to two hours for passive descent
Correct answer:
> Allow up to two hours for passive descent
The correct answer is allow up to two hours for passive descent.
Primiparous women are likely to have fewer rota onal or mid-cavity
opera ve deliveries when pushing is delayed for 1–2 hours or un l
they have a strong urge to push. Although a small trial suggested
that star ng oxytocin at full dilata on reduced the opera ve
delivery rate, NICE concluded it should not be used on the basis of
one study. See Na onal Ins tute for Health and Clincial Excellence.
Intrapartum care. CG190. London: NICE; 2014
(h ps://www.nice.org.uk/guidance/cg190) and Royal College of
Obstetricians and Gynaecologists. Opera ve vaginal delivery. Green-
top Guideline 26. London: RCOG; 2011
(h ps://www.rcog.org.uk/en/guidelines-research-
services/guidelines/gtg26/).
https://stratog.rcog.org.uk/sbas-part-2-mrcog-online-resource/obstetric-sbas/obstetrics-page-3 3/8
12/8/2018 Obstetrics page 3 | StratOG
A Your answer:
> Benzyl penicillin intravenously
Correct answer:
> Benzyl penicillin intravenously
The correct answer is benzyl penicillin intravenously. GBS
(streptococcus agalac ae) is an important cause of neonatal
morbidity and mortality. Studies have shown that both ampicillin
and benzyl penicillin reduce the incidence of early onset disease,
but benzyl penicillin is recommended because it is less likely to
promote an bio c resistance. If penicillins are contraindicated,
clindamycin is recommended unless there is evidence of local
resistance pa erns that would suggest using an alterna ve. See
Mugglestone MA, Murphy MS, Visin n C, Howe DT, Turner MA.
An bio cs for early-onset neonatal infec on: a summary of the
NICE guideline 2012. The Obstetrician & Gynaecologist 2014;16:87–
92 (h p://onlinelibrary.wiley.com/doi/10.1111/tog.12085/full).
A Your answer:
> Thick and thin blood film for parasites
Correct answer:
> Thick and thin blood film for parasites
The correct answer is thick and thin blood film for parasites. The
gold standard is thick and thin blood films in pregnancy rather than
a rapid diagnos c test. Serology is only useful in syphilis. See Royal
College of Obstetricians and Gynaecologists. The diagnosis and
https://stratog.rcog.org.uk/sbas-part-2-mrcog-online-resource/obstetric-sbas/obstetrics-page-3 4/8
12/8/2018 Obstetrics page 3 | StratOG
A Your answer:
> Another ECV with tocolysis
Correct answer:
> Another ECV with tocolysis
The correct answer is another ECV with tocolysis. ECV should be
offered a er 37 weeks of gesta on in mul parous women and a er
36 weeks of gesta on in primiparous women. Another ECV can be
offered if the first one fails. The use of tocolysis increases the
success rate a er a failed ini al a empt. If a caesarean secton is
offered it needs to be a er 38+6 weeks of gesta on. Breech delivery
may not be the most appropriate management considering she is
primiparous. There is insufficient evidence to support the use of
postural management or Moxibus on as a method of promo ng
spontaneous version over ECV. See Royal College of Obstetricians
and Gynaecologists. External cephalic version (ECV) and reducing
the incidence of breech presenta on. Green-top Guideline 20a.
London: RCOG; 2010 (h ps://www.rcog.org.uk/en/guidelines-
research-services/guidelines/gtg20a/).
https://stratog.rcog.org.uk/sbas-part-2-mrcog-online-resource/obstetric-sbas/obstetrics-page-3 5/8
12/8/2018 Obstetrics page 3 | StratOG
A Your answer:
> Perform a CTG and arrange a scan
Correct answer:
> Perform a CTG and arrange a scan
The correct answer is to perform a CTG and arrange a scan.
Counselling of women in the antenatal period about the significance
of fetal movements and rela onship of this to s ll births is
increasingly being offered in UK. Delivery would not be warranted
unless further tes ng reveals an abnormality, e.g. an abnormal
Doppler scan or a pathological CTG. There is no evidence that any
formal defini on of reduced fetal movements is of greater value
than subjec ve maternal percep on in the detec on of fetal
compromise. Biophysical profiling has not shown to be of benefit.
See Unterscheider J, Horgan R, O'Donoghue K, Greene R. Reduced
fetal movements. The Obstetrician & Gynaecologist 2009;11:245–51
(h p://onlinelibrary.wiley.com/doi/10.1576/toag.11.4.245.27527/fu
ll) and Royal College of Obstetricians and Gynaecologists. Reduced
fetal movements. Green-top Guideline 57. London: RCOG; 2011
(h ps://www.rcog.org.uk/en/guidelines-research-
services/guidelines/gtg57/).
A woman presents for booking in the first trimester, she is taking lithium for
Q her mental health. How o en should her serum lithium levels be checked?
A Your answer:
> Every 4 weeks un l 36 weeks of gesta on
Correct answer:
> Every 4 weeks un l 36 weeks of gesta on
The correct answer is every 4 weeks un l 36 weeks of gesta on.
Lithium is an important drug in maintaining mental health but
taking it in pregnancy is not without risks as the incidence of fetal
heart defects are increased. If it is not for the woman to stop taking
the drug prior to concep on, lithium levels should be monitored
every 4 weeks un l 36 weeks of gesta on, and then weekly un l
delivery. Lithium levels should be checked again within 24 hours of
delivery and the dose should be adjusted to maintain a level in the
lower part of the therapeu c range. See Na onal Ins tute for
https://stratog.rcog.org.uk/sbas-part-2-mrcog-online-resource/obstetric-sbas/obstetrics-page-3 6/8
12/8/2018 Obstetrics page 3 | StratOG
A Your answer:
> Gesta onal thrombocytopenia
Correct answer:
> Gesta onal thrombocytopenia
The correct answer is gesta onal thrombocytopenia. Gesta onal
thrombocytopaenia occurs in up to 1 in 20 pregnancies. If the count
is greater than 100 x 109/l no further inves ga ons are required but
other disorders should be considered. If the count falls below this,
further inves ga ons are indicated including blood film, coagula on
screen, renal and liver func on tests, an phospholipid an bodies
and an -DNA an bodies. See Pavord S, Fairlie F. Obstetric
haematology manual. In: Dewhurst’s textbook of obstetrics and
gynaecology, 7th edi on. Wiley-Blackwell. 2007.
https://stratog.rcog.org.uk/sbas-part-2-mrcog-online-resource/obstetric-sbas/obstetrics-page-3 7/8
12/8/2018 Obstetrics page 3 | StratOG
27 Sussex Place
Regent's Park
London NW1 4RG
UK
https://stratog.rcog.org.uk/sbas-part-2-mrcog-online-resource/obstetric-sbas/obstetrics-page-3 8/8
11/21/2018 Obstetrics page 4 | StratOG
(/)
Obstetrics page 4
Assessment Total A empts: 2 Highest Score: 100 %
History
View last results (/quiz/latest/305494/299345) View highest
result (/quiz/highest/305494/299345) Retake quiz
(/quiz/retake/299345)
Ultraviolet B phototherapy
Emollients
Oral an histamines
Oral prednisolone
https://stratog.rcog.org.uk/sbas-part-2-mrcog-online-resource/obstetric-sbas/obstetrics-page-4 1/6
11/21/2018 Obstetrics page 4 | StratOG
At the evening handover of a busy labour ward, you are informed that a cord
Q prolapse has been diagnosed a er amniotomy with the presen ng part at –3
sta on. On CTG, the baseline is 115 bpm with 10 bpm variability and one variable
decelera on las ng less than 30 seconds over the last 10 minutes. The obstetric
emergency theatre is currently being used for a manual removal of the placenta.
What is the most appropriate management for this woman?
Ask a midwife to elevate the fetal presen ng part and arrange a category 1
sec on in the second theatre
Fill up the bladder via a urinary catheter and keep woman in a knee-chest
posi on un l the case in theatre is finished
Give tocolysis and wait for the emergency theatre to become free
Open the second emergency theatre for a category 1 sec on
You have been asked to review a postnatal woman with known type 1 insulin
Q dependent diabetes mellitus who was successfully delivered overnight. She is now
ea ng and drinking normally and the postdelivery capillary blood glucose readings
are all between 4 and 7 mmol/l. The plan is to stop the intravenous
insulin/dextrose sliding scale and recommence subcutaneous insulin. She wishes
to breas eed her baby. What is the most appropriate advice for the woman
regarding recommencing her subcutaneous insulin?
Con nue on the dose of insulin she was taking prior to her induc on
https://stratog.rcog.org.uk/sbas-part-2-mrcog-online-resource/obstetric-sbas/obstetrics-page-4 2/6
11/21/2018 Obstetrics page 4 | StratOG
Cherney
Joel-Cohen
Kϋstner
Maylard
Pfannens el
Acetazolamide
Nifedepine
Propranolol
Sumatripan
0%
25%
50%
75%
https://stratog.rcog.org.uk/sbas-part-2-mrcog-online-resource/obstetric-sbas/obstetrics-page-4 4/6
11/21/2018 Obstetrics page 4 | StratOG
An 18-year-old woman is pregnant with a male fetus. She has cys c fibrosis and
Q her partner is a carrier. She is worried that the baby will inherit cys c fibrosis.
What is the likelihood that the baby will be affected?
0%
25%
50%
75%
100%
Migraine
Severe pre-eclampsia
Trigeminal neuralgia
https://stratog.rcog.org.uk/sbas-part-2-mrcog-online-resource/obstetric-sbas/obstetrics-page-4 5/6
11/21/2018 Obstetrics page 4 | StratOG
A 25-year-old woman with sickle cell disease is considering having a child with her
Q partner who has sickle cell trait. What is the probability that the child will have
sickle cell disease?
25%
33%
50%
75%
100%
This is the last ques on of this quiz. Press finish to submit your choice(s) and reveal the answer(s).
https://stratog.rcog.org.uk/sbas-part-2-mrcog-online-resource/obstetric-sbas/obstetrics-page-4 6/6
11/21/2018 Obstetrics page 4 | StratOG
(/)
Obstetrics page 4
Assessment Total A empts: 2 Highest Score: 100 %
History
View last results (/quiz/latest/305494/299345) View highest
result (/quiz/highest/305494/299345) Retake quiz
(/quiz/retake/299345)
A Your answer:
> Emollients
Correct answer:
> Emollients
The correct answer is emollients. The two most common skin
problems in pregnancy are atopic erup on of pregnancy and
polymorphic erup on of pregnancy. In about half of all women who
https://stratog.rcog.org.uk/sbas-part-2-mrcog-online-resource/obstetric-sbas/obstetrics-page-4 1/8
11/21/2018 Obstetrics page 4 | StratOG
At the evening handover of a busy labour ward, you are informed that a
Q cord prolapse has been diagnosed a er amniotomy with the presen ng
part at –3 sta on. On CTG, the baseline is 115 bpm with 10 bpm variability
and one variable decelera on las ng less than 30 seconds over the last 10
minutes. The obstetric emergency theatre is currently being used for a
manual removal of the placenta. What is the most appropriate
management for this woman?
A Your answer:
> Open the second emergency theatre for a category 2 sec on
Correct answer:
> Open the second emergency theatre for a category 2 sec on
The correct answer is to open the second emergency theatre for a
category 2 sec on. A category 2 caesarean sec on is appropriate for
women in whom the fetal heart rate pa ern is normal. However, if
the CTG becomes abnormal it should be re-categorised to category
1. See Royal College of Obstetricians and Gynaecologists. Umbilical
cord prolapse. Green-top Guideline 50. London: RCOG; 2014
(h ps://www.rcog.org.uk/en/guidelines-research-
services/guidelines/gtg50/).
You have been asked to review a postnatal woman with known type 1
Q insulin dependent diabetes mellitus who was successfully delivered
overnight. She is now ea ng and drinking normally and the postdelivery
capillary blood glucose readings are all between 4 and 7 mmol/l. The plan is
to stop the intravenous insulin/dextrose sliding scale and recommence
https://stratog.rcog.org.uk/sbas-part-2-mrcog-online-resource/obstetric-sbas/obstetrics-page-4 2/8
11/21/2018 Obstetrics page 4 | StratOG
subcutaneous insulin. She wishes to breas eed her baby. What is the most
appropriate advice for the woman regarding recommencing her
subcutaneous insulin?
A Your answer:
> Reduce her prepregnancy insulin dose by 25%
Correct answer:
> Reduce her prepregnancy insulin dose by 25%
The correct answer is to reduce her prepregnancy insulin dose by
25%. Once women with type 1 diabetes are ea ng normally,
subcutaneous insulin should be recommenced at a 25% lower dose
of her prepregnancy dose if she intends to breas eed. Breas eeding
is associated with increased energy expenditure. Nelson-Piercy C.
Handbook of obstetric medicine. Fourth edi on. CRC Press. 2010.
A Your answer:
> Joel-Cohen
Correct answer:
> Joel-Cohen
The correct answer is Joel-Cohen. Pfannens el and Kustner are
curved incisions using sharp dissec on. Cherney and Maylard are
muscle cu ng incisions. Raghavan R, Arya P, Arya P, China S.
Abdominal incisions and sutures in obstetrics and gynaecology. The
Obstetrician & Gynaecologist 2014;16:13–18
(h p://onlinelibrary.wiley.com/doi/10.1111/tog.12063/full).
https://stratog.rcog.org.uk/sbas-part-2-mrcog-online-resource/obstetric-sbas/obstetrics-page-4 3/8
11/21/2018 Obstetrics page 4 | StratOG
A Your answer:
> Acetazolamide
Correct answer:
> Acetazolamide
The correct answer is acetazolamide. Idiopathic intracranial
hypertension (IIH) is a rare but important cause of headache in
pregnancy. A detailed history and examina on is essen al. IIH tends
to present in the first half of pregnancy and women with IIH are
o en overweight. The diagnosis is made using the modified Dandy
criteria. See Thirumalaikumar L, Ramalingam K, Heafield T.
Idiopathic intracranial hypertension in pregnancy. The Obstetrician
& Gynaecologist 2014;16:93–7
(h p://onlinelibrary.wiley.com/doi/10.1111/tog.12087/full).
A Your answer:
> Urgent referral for laser abla on of the placental bed
Correct answer:
> Urgent referral for laser abla on of the placental bed
The correct answer is urgent referral for laser abla on of the
placental bed. The twins have developed twin to twin transfusion
syndrome (TTTS) due to vascular placental anastomoses which are
almost universal in monochorionic twin pregnancies. Despite the
https://stratog.rcog.org.uk/sbas-part-2-mrcog-online-resource/obstetric-sbas/obstetrics-page-4 4/8
11/21/2018 Obstetrics page 4 | StratOG
A Your answer:
> 0%
Correct answer:
> 0%
The correct answer is 0%. The pa ent’s mother must be a carrier.
She will have inherited the gene from her father. However the
pa ent’s grandmother must also be a carrier since the pa ent’s
uncle has the disease but her mother did not inherit the gene since
she is well. The pa ent has a 50% chance of being a carrier, but with
a healthy husband it is very unlikely any daughter of hers will have
the disease since she will only inherit an affected gene from her
mother unless her husband’s sperm has a new muta on.
https://stratog.rcog.org.uk/sbas-part-2-mrcog-online-resource/obstetric-sbas/obstetrics-page-4 5/8
11/21/2018 Obstetrics page 4 | StratOG
An 18-year-old woman is pregnant with a male fetus. She has cys c fibrosis
Q and her partner is a carrier. She is worried that the baby will inherit cys c
fibrosis. What is the likelihood that the baby will be affected?
A Your answer:
> 50%
Correct answer:
> 50%
The correct answer is 50%. The woman is heterozygous so will
inevitably pass on the CF gene and there is a 50% chance of her
baby acquiring the gene from her partner. The child will be either a
carrier or affected.
A Your answer:
> Idiopathic intracranial hypertension (IIH)
Correct answer:
> Idiopathic intracranial hypertension (IIH)
The correct answer is idiopathic intracranial hypertension (IHH). IHH
is a diagnosis of exclusion in a pregnant woman with a headache. It
is more wommen in women, with a female:male ra o of 8:1. IHH is
also more comment in obese women, with an incidence of 19/100
000 compared with <1/100 000 in non-obese women. Rising obesity
rates will therefore lead to an increasing incidence of IHH. See
Thirumalaikumar L, Ramalingam K, Heafield T. Idiopathic intracranial
hypertension in pregnancy. The Obstetrician & Gynaecologist
2014;16:93–97
(h p://onlinelibrary.wiley.com/doi/10.1111/tog.12087/full).
https://stratog.rcog.org.uk/sbas-part-2-mrcog-online-resource/obstetric-sbas/obstetrics-page-4 6/8
11/21/2018 Obstetrics page 4 | StratOG
A Your answer:
> 50%
Correct answer:
> 50%
The correct answer is 50%. Following screening, this couple is
iden fied as 'at risk'. They need counselling and advice about their
reproduc ve op ons, including the methods and risks of prenatal
screening and termina on of pregnancy. See Royal College of
Obstetricians and Gynaecologists. Management of sickle cell disease
in pregnancy. Green-top Guideline 61. London: RCOG; 201
(h ps://www.rcog.org.uk/en/guidelines-research-
services/guidelines/gtg61/)1.
https://stratog.rcog.org.uk/sbas-part-2-mrcog-online-resource/obstetric-sbas/obstetrics-page-4 7/8
11/21/2018 Obstetrics page 4 | StratOG
https://stratog.rcog.org.uk/sbas-part-2-mrcog-online-resource/obstetric-sbas/obstetrics-page-4 8/8
11/21/2018 Obstetrics page 5 | StratOG
(/)
Obstetrics page 5
Assessment Total A empts: 2 Highest Score: 100 %
History
View last results (/quiz/latest/305574/299371) View highest
result (/quiz/highest/305574/299371) Retake quiz
(/quiz/retake/299371)
A recently delivered woman on the postnatal ward tells you that her baby has a
Q patent ductus arteriosus. She asks what the ductus arteriosus is connected to
when her baby was in utero. Where does the ductus arteriosus connects in a
fetus?
https://stratog.rcog.org.uk/sbas-part-2-mrcog-online-resource/obstetric-sbas/obstetrics-page-5 1/7
11/21/2018 Obstetrics page 5 | StratOG
Await the onset of spontaneous labour and give the newborn varicella
zoster immunoglobulin (VZIG)
Await the onset of spontaneous labour and give the newborn varicella
zoster immunoglobulin if delivered within 7 days following the onset of the
maternal rash
Give the mother varicella zoster immunoglobulin and await the onset of
spontaneous labour
Give the mother varicella zoster immunoglobulin and induce the following
day at 39 weeks of gesta on
Induce labour the following day at 39 weeks of gesta on and give the
newborn varicella zoster immunoglobulin
https://stratog.rcog.org.uk/sbas-part-2-mrcog-online-resource/obstetric-sbas/obstetrics-page-5 2/7
11/21/2018 Obstetrics page 5 | StratOG
A 19-year-old woman is 28 weeks into her first pregnancy. On rou ne blood tests,
Q her haemoglobin is 95 g/l. What is the best test to diagnose iron deficiency
anaemia?
Blood film
Serum ferri n
Serum iron levels
31–40%
41–50%
51–60%
61–70%
71–80%
https://stratog.rcog.org.uk/sbas-part-2-mrcog-online-resource/obstetric-sbas/obstetrics-page-5 3/7
11/21/2018 Obstetrics page 5 | StratOG
Graves disease
Hashimoto thyroidi s
Subacute thyroidi s
You see a woman who is 35 weeks pregnant in your day assessment unit. She
Q presents with nausea, anorexia and generalised malaise. Her liver func on test
demonstrates an alanine transaminase (ALT) of 634. Which of the following
features is most useful in dis nguishing acute fa y liver of pregnancy (AFLP) from
HELLP syndrome?
Hypertension
Hypoglycaemia
Proteinuria
https://stratog.rcog.org.uk/sbas-part-2-mrcog-online-resource/obstetric-sbas/obstetrics-page-5 4/7
11/21/2018 Obstetrics page 5 | StratOG
You see a woman who is 35 weeks pregnant in your day assessment unit. She
Q presents with itching. Your differen al diagnosis is obstetric cholestasis. Your ST1
asks you if she should prescribe vitamin K but is not sure how it works. Vitamin K is
responsible for manufacturing which of the following coagula on factors?
Factor V
Factor VIII
Factor X
Factor XI
Factor XII
You see a woman who is 35 weeks pregnant in your day assessment unit. She
Q presents with itching. Your differen al diagnosis is polymorphic erup on of
pregnancy. What clinical feature is most helpful in diagnosing this condi on?
Facial pigmenta on
Umbilical rash
https://stratog.rcog.org.uk/sbas-part-2-mrcog-online-resource/obstetric-sbas/obstetrics-page-5 5/7
11/21/2018 Obstetrics page 5 | StratOG
You see a woman who is 35 weeks pregnant in your day assessment unit. She
Q presents with itching causing insomnia of the palms of hands and soles of feet.
There are scratch marks but no rash. Her alanine transaminase is 78 IU/l (normal
range 10–35) and bile acids are 42 micromol/l (normal range 1–10). Which of the
following contracep ves should be avoided postnatally?
Condoms
Depo Provera®
This is the last ques on of this quiz. Press finish to submit your choice(s) and reveal the answer(s).
https://stratog.rcog.org.uk/sbas-part-2-mrcog-online-resource/obstetric-sbas/obstetrics-page-5 6/7
11/21/2018 Obstetrics page 5 | StratOG
https://stratog.rcog.org.uk/sbas-part-2-mrcog-online-resource/obstetric-sbas/obstetrics-page-5 7/7
11/21/2018 Obstetrics page 5 | StratOG
(/)
Obstetrics page 5
Assessment Total A empts: 2 Highest Score: 100 %
History
View last results (/quiz/latest/305574/299371) View highest
result (/quiz/highest/305574/299371) Retake quiz
(/quiz/retake/299371)
A recently delivered woman on the postnatal ward tells you that her baby
Q has a patent ductus arteriosus. She asks what the ductus arteriosus is
connected to when her baby was in utero. Where does the ductus
arteriosus connects in a fetus?
A Your answer:
> Pulmonary artery to aorta
Correct answer:
> Pulmonary artery to aorta
The correct answer is the pulmonary artery to the aorta. An
understanding of fetal circula on and congenital heart defects is
important to an obstetrician. It gives them the ability to discuss any
https://stratog.rcog.org.uk/sbas-part-2-mrcog-online-resource/obstetric-sbas/obstetrics-page-5 1/7
11/21/2018 Obstetrics page 5 | StratOG
A Your answer:
> Approximately 50% of newly diagnosed pa ents have no family
history
Correct answer:
> Approximately 50% of newly diagnosed pa ents have no family
history
The correct answer is approximately 50% of newly diagnosed
pa ents have no family history. Daughters of affected males will
always be carriers but sons will never inherit the disease (the
affected gene is on the paternal X chromosome, which never goes
to the sons). Haemophilia can arise as a spontaneous muta on and
the risk of being a carrier is 1 in 20 000. See Mumford A. Gene c
counselling and pre-natal diagnosis. In: Pavord S, Hunt B (editors).
The obstetric haematology manual. Cambridge University Press.
2010. p 194–199.
https://stratog.rcog.org.uk/sbas-part-2-mrcog-online-resource/obstetric-sbas/obstetrics-page-5 2/7
11/21/2018 Obstetrics page 5 | StratOG
A Your answer:
> Await the onset of spontaneous labour and give the newborn
varicella zoster immunoglobulin if delivered within 7 days
following the onset of the maternal rash
Correct answer:
> Await the onset of spontaneous labour and give the newborn
varicella zoster immunoglobulin if delivered within 7 days
following the onset of the maternal rash
The correct answer is await the onset of spontaneous labour and
give the newborn varicella zoster immunoglobulin if delivered
within 7 days following the onset of the maternal rash. VZIG has no
effect once chickenpox has developed. If the woman presents within
24 hours (at over 20 weeks of gesta on) it is worth prescribing
acyclovir. The baby is at most risk if delivered within a week of the
development of the infec on. A er 7 days the maternal an bodies
will protect the baby. See Royal College of Obstetricians and
Gynaecologists. Chickenpox in pregnancy. Green-top Guideline 13.
London: RCOG; 2007 (h ps://www.rcog.org.uk/en/guidelines-
research-services/guidelines/gtg13/).
A Your answer:
> Serum ferri n
Correct answer:
> Serum ferri n
The correct answer is serum ferri n. Although an approxima on of
iron deficiency can be assessed by the mean corpuscular volume,
serum ferri n will give an accurate test of iron stores. See Bri sh
Commi ee for Standards in Haematology. UK guidelines on the
management of iron deficiency in pregnancy. London: BCSH: 2011
(h p://www.bcshguidelines.com/documents/UK_Guidelines_iron_d
eficiency_in_pregnancy.pdf).
https://stratog.rcog.org.uk/sbas-part-2-mrcog-online-resource/obstetric-sbas/obstetrics-page-5 3/7
11/21/2018 Obstetrics page 5 | StratOG
A Your answer:
> 61–70%
Correct answer:
> 61–70%
The correct answer is 61–70%. Induc on of labour should only be
offered to women in specific circumstances since there is an
increased risk of caesarean sec on. See Na onal Ins tute for Health
and Clinical Excellence. Induc on of labour. Clinical guideline 70.
London: NICE. 2008 (h ps://www.nice.org.uk/guidance/cg70).
A Your answer:
> Graves disease
Correct answer:
> Graves disease
The correct answer is Graves disease. 95% of cases of
hyperthyroidism in pregnancy are due to Graves disease. Thyroxine
produc on increases in pregnancy due to an increase in thyroxine
binding globulin to maintain a steady free thyroxine level (both T3
and T4). In assessing thyroid func on in pregnancy, free T3 and T4
levels reflect thyroid func on rather than total T3 and T4 levels. In
monitoring hypo- and hyperthyroid disease the TSH level may take
longer to return to normal so free T3 and T4 levels are a more
accurate reflec on. Hyperthyroidism is common in women of
reproduc ve years and is seen in approximately 1 in 500
pregnancies. See Nelson-Piercy C. Handbook of obstetric medicine,
4th edi on. CRC Press. 2010.
https://stratog.rcog.org.uk/sbas-part-2-mrcog-online-resource/obstetric-sbas/obstetrics-page-5 4/7
11/21/2018 Obstetrics page 5 | StratOG
You see a woman who is 35 weeks pregnant in your day assessment unit.
Q She presents with nausea, anorexia and generalised malaise. Her liver
func on test demonstrates an alanine transaminase (ALT) of 634. Which of
the following features is most useful in dis nguishing acute fa y liver of
pregnancy (AFLP) from HELLP syndrome?
A Your answer:
> Hypoglycaemia
Correct answer:
> Hypoglycaemia
The correct answer is hypoglycaemia. Liver disorders are common in
pregnancy, but rarely cause long term problems. AFLP is a rare but
serious condi on which will share many common features with
HELLP. However hypoglycaemia is common in AFLP and can be
severe, but is extremely unlikely in HELLP. See Nelson-Piercy C.
Handbook of obstetric management, 4th edi on. CRC Press. 2010.
You see a woman who is 35 weeks pregnant in your day assessment unit.
Q She presents with itching. Your differen al diagnosis is obstetric cholestasis.
Your ST1 asks you if she should prescribe vitamin K but is not sure how it
works. Vitamin K is responsible for manufacturing which of the following
coagula on factors?
A Your answer:
> Factor X
Correct answer:
> Factor X
The correct answer is factor X. Vitamin K is required for
manufacturing coagula on factors II, VII, IX, X. See Royal College of
Obstetricians and Gynaecologists. Obstetric cholestasis. Green-top
Guideline 43. London: RCOG; 2011
(h ps://www.rcog.org.uk/en/guidelines-research-
services/guidelines/gtg43/).
https://stratog.rcog.org.uk/sbas-part-2-mrcog-online-resource/obstetric-sbas/obstetrics-page-5 5/7
11/21/2018 Obstetrics page 5 | StratOG
You see a woman who is 35 weeks pregnant in your day assessment unit.
Q She presents with itching. Your differen al diagnosis is polymorphic
erup on of pregnancy. What clinical feature is most helpful in diagnosing
this condi on?
A Your answer:
> Inflamed abdominal striae
Correct answer:
> Inflamed abdominal striae
The correct answer is inflamed abdominal striae. Polymorphic
erup on of pregnancy classically affects the abdominal striae,
sparing the umbilicus. The differen al diagnosis is intrahepa c
cholestasis of pregnancy, atopic erup on of pregnancy and
pemphigoid gesta onis. See Nelson-Piercy C. Handbook of obstetric
management, 4th edi on. CRC Press 2010 and Maharajan A, Aye C,
Ratnavel R, Burova E. Skin erup ons specific to pregnancy: an
overview. The Obstetrician & Gynaecologist 2013;15:233–40
(h p://onlinelibrary.wiley.com/doi/10.1111/tog.12051/full).
You see a woman who is 35 weeks pregnant in your day assessment unit.
Q She presents with itching causing insomnia of the palms of hands and soles
of feet. There are scratch marks but no rash. Her alanine transaminase is 78
IU/l (normal range 10–35) and bile acids are 42 micromol/l (normal range
1–10). Which of the following contracep ves should be avoided
postnatally?
A Your answer:
> Combined oral contracep ve pill
Correct answer:
> Combined oral contracep ve pill
The correct answer is the combined oral contracep ve pill.
Estrogen-containing contracep ves should be avoided in women
who have had obstetric cholestasis. See Royal College of
https://stratog.rcog.org.uk/sbas-part-2-mrcog-online-resource/obstetric-sbas/obstetrics-page-5 6/7
11/21/2018 Obstetrics page 5 | StratOG
https://stratog.rcog.org.uk/sbas-part-2-mrcog-online-resource/obstetric-sbas/obstetrics-page-5 7/7
11/21/2018 Obstetrics page 6 | StratOG
(/)
Obstetrics page 6
Assessment Total A empts: 2 Highest Score: 100 %
History
View last results (/quiz/latest/304860/301380) View highest
result (/quiz/highest/304860/301380) Retake quiz
(/quiz/retake/301380)
https://stratog.rcog.org.uk/sbas-part-2-mrcog-online-resource/obstetric-sbas/obstetrics-page-6 1/6
11/21/2018 Obstetrics page 6 | StratOG
Protein S deficiency
Prothrombin muta on heterozygosity
1000–1499 ml
1500–1999 ml
2000–2499 ml
2500–2999 ml
3000–3499 ml
https://stratog.rcog.org.uk/sbas-part-2-mrcog-online-resource/obstetric-sbas/obstetrics-page-6 2/6
11/21/2018 Obstetrics page 6 | StratOG
A pregnant woman with severe von Willebrand’s disease a ends the antenatal
Q clinic. She is nonsensi sed Rh-nega ve. What is the recommend management
regarding rou ne antenatal an -D prophylaxis?
Administer intravenous an -D
Administer oral an -D
Do not administer an -D
Maternal smoking
Previous myomectomy
https://stratog.rcog.org.uk/sbas-part-2-mrcog-online-resource/obstetric-sbas/obstetrics-page-6 3/6
11/21/2018 Obstetrics page 6 | StratOG
Hepatosplenomegaly
Meconium ileus
Pneumonia
Polycythaemia
Lanz
Low transverse
Lower midline
Upper midline
A 30-year-old woman, Para 0, is referred for a growth scan. The pregnancy has
Q been uncomplicated so far. The ultrasonographer reports that the es mated fetal
weight is on the 5th cen le for gesta on, there is normal liquor and the umbilical
https://stratog.rcog.org.uk/sbas-part-2-mrcog-online-resource/obstetric-sbas/obstetrics-page-6 4/6
11/21/2018 Obstetrics page 6 | StratOG
artery Doppler waveform is normal but the fetal head circumference is less than
the 1st cen le for gesta on. What is the most likely infec ve cause?
Cytomegalovirus
Epstein–Barr virus
Rubella
Syphilis
Toxoplasmosis
Cranial ultrasound
CT scan
MR venogram
Positron emission tomography scan
Skull x-ray
https://stratog.rcog.org.uk/sbas-part-2-mrcog-online-resource/obstetric-sbas/obstetrics-page-6 5/6
11/21/2018 Obstetrics page 6 | StratOG
3 days
4 days
5 days
6 days
7 days
This is the last ques on of this quiz. Press finish to submit your choice(s) and reveal the answer(s).
https://stratog.rcog.org.uk/sbas-part-2-mrcog-online-resource/obstetric-sbas/obstetrics-page-6 6/6
11/21/2018 Obstetrics page 6 | StratOG
(/)
Obstetrics page 6
Assessment Total A empts: 2 Highest Score: 100 %
History
View last results (/quiz/latest/304860/301380) View highest
result (/quiz/highest/304860/301380) Retake quiz
(/quiz/retake/301380)
A Your answer:
> Increased specificity of the test
Correct answer:
> Increased specificity of the test
https://stratog.rcog.org.uk/sbas-part-2-mrcog-online-resource/obstetric-sbas/obstetrics-page-6 1/7
11/21/2018 Obstetrics page 6 | StratOG
A Your answer:
> Factor V Leiden homozygosity
Correct answer:
> Factor V Leiden homozygosity
The correct answer is Factor V Leiden homozygosity.
A Your answer:
> 2000–2499 ml
Correct answer:
> 2000–2499 ml
The correct answer is 2000–2499 ml. This describes a class III
haemorrhage, i.e. 30–40% of circula ng volume lost (pulse
>120<140, BP decreased, agitated and confused mental state).
https://stratog.rcog.org.uk/sbas-part-2-mrcog-online-resource/obstetric-sbas/obstetrics-page-6 2/7
11/21/2018 Obstetrics page 6 | StratOG
A Your answer:
> Administer intravenous an -D
Correct answer:
> Administer intravenous an -D
The answer is administer intravenous an -D.
A Your answer:
> Maternal age of more than 40 years
Correct answer:
> Maternal age of more than 40 years
The answer is maternal age of more than 40 years, as it is associated
with a ninefold risk of placental praevia.
https://stratog.rcog.org.uk/sbas-part-2-mrcog-online-resource/obstetric-sbas/obstetrics-page-6 3/7
11/21/2018 Obstetrics page 6 | StratOG
A Your answer:
> Polycythaemia
Correct answer:
> Polycythaemia
The correct answer is polycythaemia.
A Your answer:
> Lower midline
Correct answer:
> Lower midline
The correct answer is lower midline.
https://stratog.rcog.org.uk/sbas-part-2-mrcog-online-resource/obstetric-sbas/obstetrics-page-6 4/7
11/21/2018 Obstetrics page 6 | StratOG
A Your answer:
> Cytomegalovirus
Correct answer:
> Cytomegalovirus
The answer is cytomegalovirus.
van Zuylen WJ, Hamilton ST, Naing Z, Hall B, Shand A, Rawlinson WD.
Congenital cytomegalovirus infec on: Clinical presenta on,
epidemiology, diagnosis and preven on. Obstet Med 2014;7:140–6.
(h ps://academic.oup.com/humrep/ar cle/15/11/2433/635079/The-
risks-associated-with-pregnancy-in-women-aged)
A Your answer:
> MR venogram
https://stratog.rcog.org.uk/sbas-part-2-mrcog-online-resource/obstetric-sbas/obstetrics-page-6 5/7
11/21/2018 Obstetrics page 6 | StratOG
Correct answer:
> MR venogram
The answer is MR venogram.
A Your answer:
> 5 days
Correct answer:
> 5 days
The answer is 5 days. When the lesions have crusted over, the
individual is no longer infec ous.
https://stratog.rcog.org.uk/sbas-part-2-mrcog-online-resource/obstetric-sbas/obstetrics-page-6 6/7
11/21/2018 Obstetrics page 6 | StratOG
27 Sussex Place
Regent's Park
London NW1 4RG
UK
https://stratog.rcog.org.uk/sbas-part-2-mrcog-online-resource/obstetric-sbas/obstetrics-page-6 7/7
11/21/2018 Obstetrics page 7 | StratOG
(/)
Obstetrics page 7
Assessment Total A empts: 2 Highest Score: 100 %
History
View last results (/quiz/latest/306302/301524) View highest
result (/quiz/highest/306302/301524) Retake quiz
(/quiz/retake/301524)
Age
Anaemia
BMI
Gesta onal diabetes
Sore throat
https://stratog.rcog.org.uk/sbas-part-2-mrcog-online-resource/obstetric-sbas/obstetrics-page-7 1/4
11/21/2018 Obstetrics page 7 | StratOG
A pregnant woman who is known to have poorly controlled epilepsy is found dead
Q at her home at 22 weeks of gesta on. According to the MBRRACE 2014 report,
what is the most likely cause of her death?
Cerebrovascular accident
Eclamp c seizure
https://stratog.rcog.org.uk/sbas-part-2-mrcog-online-resource/obstetric-sbas/obstetrics-page-7 2/4
11/21/2018 Obstetrics page 7 | StratOG
A woman with a spinal cord transec on presents in labour at term. She is having
Q regular, strong uterine contrac ons but does not experience any pain. What is the
level of her spinal cord injury?
L1
L2
T8
T10
T12
This is the last ques on of this quiz. Press finish to submit your choice(s) and reveal the answer(s).
https://stratog.rcog.org.uk/sbas-part-2-mrcog-online-resource/obstetric-sbas/obstetrics-page-7 3/4
11/21/2018 Obstetrics page 7 | StratOG
https://stratog.rcog.org.uk/sbas-part-2-mrcog-online-resource/obstetric-sbas/obstetrics-page-7 4/4
11/21/2018 Obstetrics page 7 | StratOG
(/)
Obstetrics page 7
Assessment Total A empts: 2 Highest Score: 100 %
History
View last results (/quiz/latest/306302/301524) View highest
result (/quiz/highest/306302/301524) Retake quiz
(/quiz/retake/301524)
A Your answer:
> Age
Correct answer:
> Age
The answer is her age.
https://stratog.rcog.org.uk/sbas-part-2-mrcog-online-resource/obstetric-sbas/obstetrics-page-7 1/4
11/21/2018 Obstetrics page 7 | StratOG
A Your answer:
> Immediate treatment with insulin and/or me ormin
Correct answer:
> Immediate treatment with insulin and/or me ormin
The answer is immediate treatment with insulin and/or me ormin.
Na onal Ins tute for Health and Care Excellence. 1.2 Gesta onal
diabetes. In: Diabetes in Pregnancy: Management from
Preconcep on to the Postnatal Period.NG3. London: NICE; 2015.
(h ps://www.nice.org.uk/guidance/ng3/chapter/1-
recommenda ons#gesta onal-diabetes-2)
A Your answer:
> Sudden unexplained death in pregnancy
Correct answer:
> Sudden unexplained death in pregnancy
https://stratog.rcog.org.uk/sbas-part-2-mrcog-online-resource/obstetric-sbas/obstetrics-page-7 2/4
11/21/2018 Obstetrics page 7 | StratOG
A Your answer:
> T10
Correct answer:
> T10
The answer is T10.
© 2018
https://stratog.rcog.org.uk/sbas-part-2-mrcog-online-resource/obstetric-sbas/obstetrics-page-7 4/4
11/21/2018 Gynaecology page 1 | StratOG
(/)
Gynaecology page 1
Assessment Total A empts: 2 Highest Score: 100 %
History
View last results (/quiz/latest/305518/299346) View highest
result (/quiz/highest/305518/299346) Retake quiz
(/quiz/retake/299346)
Femoral
Ilio-inguinal
Obturator
Scia c
https://stratog.rcog.org.uk/sbas-part-2-mrcog-online-resource/gynaecology-sbas/gynaecology-page-1 1/6
11/21/2018 Gynaecology page 1 | StratOG
You have been reviewing the NICE guidelines on urinary incon nence. You have
Q been asked to perform an audit on management of urinary incon nence in your
department. What is the main purpose of audit?
Changing prac ce
Collec ng data
A 47-year-old woman seeks advice about con nuing the combined oral
Q contracep ve pill (COCP). She is normotensive and a non-smoker with a BMI of 25.
She has no other medical history and no significant family history. She is
concerned that the COCP may give her addi onal health risks. Which of the
following malignancies would you advise she may have a small addi onal risk of
developing due to taking the COCP?
Breast cancer
Colorectal cancer
Endometrial cancer
Lung cancer
Ovarian cancer
https://stratog.rcog.org.uk/sbas-part-2-mrcog-online-resource/gynaecology-sbas/gynaecology-page-1 2/6
11/21/2018 Gynaecology page 1 | StratOG
A 65-year-old had a hysterectomy for endometrial cancer. She recovered well but
Q complained of dribbling urine 2 days later and was given a course of an bio cs for
a presumed UTI. On review at 4 weeks she complains of con nued urinary
incon nence. She has no dysuria, no sensa on of urgency, needs to wear a pad at
night, and intermi ently voids good volumes of urine with normal flow. Urinalysis
is nega ve. What the most likely diagnosis?
Fistula
Bowel
Nerve
Ovary
Urinary tract
Vascular
https://stratog.rcog.org.uk/sbas-part-2-mrcog-online-resource/gynaecology-sbas/gynaecology-page-1 3/6
11/21/2018 Gynaecology page 1 | StratOG
Dysmenorrhoea
You are asked to review a 55-year-old woman with overac ve bladder symptoms.
Q She has responded poorly to bladder training and is on oxybutynin therapy. Her
main complaint is nocturia, which is badly affec ng her quality of life. What is the
best treatment for her con nuing symptoms?
https://stratog.rcog.org.uk/sbas-part-2-mrcog-online-resource/gynaecology-sbas/gynaecology-page-1 4/6
11/21/2018 Gynaecology page 1 | StratOG
Darifenacin
Desmopressin
Mirabegrone
Tolterodine
Transdermal oxybutynin
This analysis below is taken from a meta-analysis of ovula on rates in women with
Q polycys c ovarian syndrome (PCOS) taking me ormin compared with clomifene
ovula on induc on therapy. Subgroup analysis was also carried out using a cut-off
BMI level of 30 kg/m2.
Me ormin is less effec ve than clomifene in the obese group (BMI >30)
https://stratog.rcog.org.uk/sbas-part-2-mrcog-online-resource/gynaecology-sbas/gynaecology-page-1 5/6
11/21/2018 Gynaecology page 1 | StratOG
Me ormin is more effec ve than clomifene in the obese group (BMI >30)
This is the last ques on of this quiz. Press finish to submit your choice(s) and reveal the answer(s).
https://stratog.rcog.org.uk/sbas-part-2-mrcog-online-resource/gynaecology-sbas/gynaecology-page-1 6/6
11/21/2018 Gynaecology page 1 | StratOG
(/)
Gynaecology page 1
Assessment Total A empts: 2 Highest Score: 100 %
History
View last results (/quiz/latest/305518/299346) View highest
result (/quiz/highest/305518/299346) Retake quiz
(/quiz/retake/299346)
A Your answer:
> Scia c
Correct answer:
> Scia c
https://stratog.rcog.org.uk/sbas-part-2-mrcog-online-resource/gynaecology-sbas/gynaecology-page-1 1/7
11/21/2018 Gynaecology page 1 | StratOG
You have been reviewing the NICE guidelines on urinary incon nence. You
Q have been asked to perform an audit on management of urinary
incon nence in your department. What is the main purpose of audit?
A Your answer:
> Improving quality
Correct answer:
> Improving quality
The correct answer is improving quality. Clinical audit is a quality
improvement process that seeks to improve pa ent care and
outcomes through systema c review of care against explicit criteria
and the implementa on of change. Aspects of the structure,
processes and outcomes of care are selected and systema cally
evaluated against explicit criteria. Where indicated, changes are
implemented at an individual, team or service level and further
monitoring is used to confirm improvement in healthcare delivery.
See Royal College of Obstetricians and Gynaecologists.
Understanding audit. Clinical Governance Advice 5. London: RCOG;
2003 (h ps://www.rcog.org.uk/en/guidelines-research-
services/guidelines/clinical-governance-advice-5/).
A 47-year-old woman seeks advice about con nuing the combined oral
Q contracep ve pill (COCP). She is normotensive and a non-smoker with a
BMI of 25. She has no other medical history and no significant family
history. She is concerned that the COCP may give her addi onal health
risks. Which of the following malignancies would you advise she may have a
small addi onal risk of developing due to taking the COCP?
https://stratog.rcog.org.uk/sbas-part-2-mrcog-online-resource/gynaecology-sbas/gynaecology-page-1 2/7
11/21/2018 Gynaecology page 1 | StratOG
A Your answer:
> Breast cancer
Correct answer:
> Breast cancer
The correct answer is breast cancer. COCP use provides a protec ve
effect against ovarian and endometrial cancer that con nues for 15
years or more a er stopping the pill. Women can be advised that
there may be a small addi onal risk of developing breast cancer if
they use COCP, which reduces to no risk 10 years a er stopping the
pill. See Faculty of Sexual & Reproduc ve Healthcare. Contracep on
for women aged over 40 years. Clinical Guidance. London: FSRH;
2010 (h p://www.fsrh.org/pdfs/Contracep onOver40July10.pdf).
A Your answer:
> Fistula
Correct answer:
> Fistula
The correct answer is fistula. In the developed world the majority of
urinary tract fistulae occur following hysterectomy (both vaginal and
abdominal) and caesarean sec on. This is usually due to failure to
dissect the bladder free of the cervix and upper vagina. Leakage
star ng in the immediate postopera ve period suggests direct
damage. Leakage that starts 1-2 weeks postopera vely is due to
avascular necrosis. See Monaghan JM, Lopes T, Naik R. Bonney's
gynaecological surgery. Wiley-Blackwell. 2004.
https://stratog.rcog.org.uk/sbas-part-2-mrcog-online-resource/gynaecology-sbas/gynaecology-page-1 3/7
11/21/2018 Gynaecology page 1 | StratOG
A Your answer:
> Urinary tract
Correct answer:
> Urinary tract
The correct answer is urinary tract injury. Laparoscopic surgery
involves risks to bowel, urinary tract and major blood vessels. These
risks are higher in women who are obese or significantly
underweight, however the risks of laparotomy are significantly
greater in the morbidly obese. Urinary tract injury and vaginal cuff
dehiscence are more common in the laparoscopic approach with an
odds ra o of 2.61 for urinary tract injury. Royal College of
Obstetricians and Gynaecologists. Preven ng entry-related
gynaecological laparoscopic injuries. Green-top Guideline 49.
London: RCOG; 2008. (h ps://www.rcog.org.uk/en/guidelines-
research-services/guidelines/gtg49/)
A Your answer:
> Failure of previous medical therapy
Correct answer:
> Failure of previous medical therapy
The correct answer is failure of previous medical therapy. An
endometrial biopsy should be taken if there is persistent
intermenstrual bleeding or if treatment is ineffec ve in women over
45. An ultrasound is the first line diagnos c tool for iden fying
structural abnormali es and should be performed if the uterus is
palpable abdominally, vaginal examina on reveals a pelvic mass or if
https://stratog.rcog.org.uk/sbas-part-2-mrcog-online-resource/gynaecology-sbas/gynaecology-page-1 4/7
11/21/2018 Gynaecology page 1 | StratOG
drug treatment fails. See Na onal Ins tute for Health and Clinical
Excellence. Heavy menstrual bleeding. London: NICE; 2013
(h ps://www.nice.org.uk/guidance/QS47).
A Your answer:
> 4 opera ons in every 100
Correct answer:
> 4 opera ons in every 100
The correct answer is 4 opera ons in every 100. The overall risk of
serious complica ons from abdominal hysterectomy is
approximately four women in every 100 (common). See Na onal
Ins tute of Health and Clinical Excellence. Venous
thromboembolism: reducing the risk. Clinical Guideline 92. London:
NICE; 2010 (h ps://www.nice.org.uk/guidance/CG92) and Royal
College of Obstetricians and Gynaecologists. Abdominal
hysterectomy for benign condi ons. Consent Advice 4. London:
RCOG; 2009 (h ps://www.rcog.org.uk/en/guidelines-research-
services/guidelines/consent-advice-4/).
A Your answer:
> Desmopressin
Correct answer:
> Desmopressin
https://stratog.rcog.org.uk/sbas-part-2-mrcog-online-resource/gynaecology-sbas/gynaecology-page-1 5/7
11/21/2018 Gynaecology page 1 | StratOG
A Your answer:
> Me ormin is less effec ve than clomifene in the obese
group (BMI >30)
Correct answer:
> Me ormin is less effec ve than clomifene in the obese
group (BMI >30)
The correct answer is that me ormin is less effec ve than clomifene
in the obese group (BMI >30). Me ormin is less effec ve than
clomifene in the obese group (BMI >30 kg/m2) as the OR = 0.43
https://stratog.rcog.org.uk/sbas-part-2-mrcog-online-resource/gynaecology-sbas/gynaecology-page-1 6/7
11/21/2018 Gynaecology page 1 | StratOG
https://stratog.rcog.org.uk/sbas-part-2-mrcog-online-resource/gynaecology-sbas/gynaecology-page-1 7/7
11/21/2018 Gynaecology page 2 | StratOG
(/)
Gynaecology page 2
Assessment Total A empts: 2 Highest Score: 100 %
History
View last results (/quiz/latest/305520/299347) View highest
result (/quiz/highest/305520/299347) Retake quiz
(/quiz/retake/299347)
https://stratog.rcog.org.uk/sbas-part-2-mrcog-online-resource/gynaecology-sbas/gynaecology-page-2 1/6
11/21/2018 Gynaecology page 2 | StratOG
A 23-year-old woman whose mother died at the age of 56 of cervical cancer comes
Q to see you. She wants to know how to reduce her own risk of cervical cancer. What
is the single most important piece of advice you could give her?
To stop smoking
Femoral
Genito-femoral
Ilio-inguinal
https://stratog.rcog.org.uk/sbas-part-2-mrcog-online-resource/gynaecology-sbas/gynaecology-page-2 2/6
11/21/2018 Gynaecology page 2 | StratOG
A 16-year-old girl presents to the gynaecology outpa ent clinic with primary
Q amenorrhea. She is 148 cm tall and weighs 54 kg (BMI 24.7). Breast development
is assessed as Tanner stage 2 and her pubic hair is noted to be sparse. Further
examina on iden fies cubitus valgus. She has no other dysmorphic features. What
is the most likely diagnosis?
Down syndrome
Mayer-Rockitansky-Kusterhauser syndrome
Turner syndrome
A 22-year-old medical student presents with a request for contracep on. Her
Q menstrual cycle is irregular and she complains of acne and hirsu sm. Previous
inves ga on has diagnosed polycys c ovary syndrome (PCOS). She wishes to have
a combined oral contracep ve with the best risk profile and most impact on her
androgenic symptoms. Which one of the following is the best available op on to
recommend for her?
https://stratog.rcog.org.uk/sbas-part-2-mrcog-online-resource/gynaecology-sbas/gynaecology-page-2 3/6
11/21/2018 Gynaecology page 2 | StratOG
1 in 50
1 in 100
1 in 500
1 in 1000
1 in 5000
1/1 to 1/10
1/10 to 1/100
1/100 to 1/1000
1/1000 to 1/10 000
https://stratog.rcog.org.uk/sbas-part-2-mrcog-online-resource/gynaecology-sbas/gynaecology-page-2 5/6
11/21/2018 Gynaecology page 2 | StratOG
10%
25%
40%
55%
70%
This is the last ques on of this quiz. Press finish to submit your choice(s) and reveal the answer(s).
https://stratog.rcog.org.uk/sbas-part-2-mrcog-online-resource/gynaecology-sbas/gynaecology-page-2 6/6
11/21/2018 Gynaecology page 2 | StratOG
(/)
Gynaecology page 2
Assessment Total A empts: 2 Highest Score: 100 %
History
View last results (/quiz/latest/305520/299347) View highest
result (/quiz/highest/305520/299347) Retake quiz
(/quiz/retake/299347)
A Your answer:
> Arrange a repeat scan a er 7 days
Correct answer:
> Arrange a repeat scan a er 7 days
https://stratog.rcog.org.uk/sbas-part-2-mrcog-online-resource/gynaecology-sbas/gynaecology-page-2 1/8
11/21/2018 Gynaecology page 2 | StratOG
A Your answer:
> To a end regularly for cervical screening
Correct answer:
> To a end regularly for cervical screening
The correct asnwer is to a end regularly for cervical screening. The
incidence of cervical carcinoma has dras cally reduced in countries
with screening programmes. Only 1% of abnormal smears progress
to malignancy over a long period of me. Most women with cervical
cancer have not had a smear in the last 5 years and many of then
have never had a smear. See Centres for Disease Control and
Preven on. Gynecologic cancers. Accessed online July 2015
(h p://www.cdc.gov/cancer/cervical/basic_info/preven on.htm).
https://stratog.rcog.org.uk/sbas-part-2-mrcog-online-resource/gynaecology-sbas/gynaecology-page-2 2/8
11/21/2018 Gynaecology page 2 | StratOG
A Your answer:
> Femoral
Correct answer:
> Femoral
The correct answer is the femoral nerve. Gynaecological surgery,
especially abdominal hysterectomy, is the most common cause of
iatrogenic femoral nerve injury, and injury to the femoral nerve is
the most common nerve injury in gynaecological prac ce. This is
usually caused by compression of the nerve against the pelvic
sidewall by a retractor blade. See Kuponiyi O, Alleemudder DI,
Latunde-Dada A, Eedarapalli P. Nerve injuries associated with
gynaecological surgery. The Obstetrician & Gynaecologist
2014;16:29–36
(h p://onlinelibrary.wiley.com/doi/10.1111/tog.12064/full).
A Your answer:
> Turner syndrome
Correct answer:
> Turner syndrome
The correct answer is Turner syndrome. The karyotype is 45 XO in
Turner syndrome. It is the most common cause of gonadal
dysgenesis. These pa ents may have addi onal renal and cardiac
anamolies. Some women may menstruate due to mosaicism, but
premature ovarian failure is more common. See Bondy CA, and for
The Turner Syndrome Consensus Study Group. Care of girls and
women with Turner syndrome: a guideline of the Turner Syndrome
Study Group. J Clin Endocrinol Metab 2007;92:10–25
(h p://press.endocrine.org/doi/abs/10.1210/jc.2006-1374?
url_ver=Z39.88-
2003&rfr_id=ori:rid:crossref.org&rfr_dat=cr_pub%3Dpubmed).
https://stratog.rcog.org.uk/sbas-part-2-mrcog-online-resource/gynaecology-sbas/gynaecology-page-2 3/8
11/21/2018 Gynaecology page 2 | StratOG
A Your answer:
> Yasmin® (ethinyl estradiol/drosperinone)
Correct answer:
> Yasmin® (ethinyl estradiol/drosperinone)
The correct answer is Yasmin® (ethinyl estradiol/drosperinone).
From the given list, Yasmin is more beneficial in terms of
management of acne and hirsui sm associated with PCOS. Women
with PCOS may also be given Marvelon or Mercilon as
contracep on. Yasmin contains 3 mg of drosperinone, which has
some an androgenic proper es. Diane e is also useful as it
contains cyproterone acetate, which is also an an androgenic agent.
Care must be taken for women with high body mass index. See
Swingler R, Awala A, Gordon U. Hirsu sm in young women. The
Obstetrician & Gynaecologist 2009;11:101–7
(h p://onlinelibrary.wiley.com/doi/10.1576/toag.11.2.101.27483/fu
ll).
A Your answer:
> 1 in 500
Correct answer:
> 1 in 500
The correct is 1 in 500. Uterine perfora on is uncommon, but a
small postmenopausal uterus is an independent risk factor,
especially if the cervical os is stenosed. The overall risk is reported
as 0.76%. See Shakir F, Diab Y. The perforated uterus. The
https://stratog.rcog.org.uk/sbas-part-2-mrcog-online-resource/gynaecology-sbas/gynaecology-page-2 4/8
11/21/2018 Gynaecology page 2 | StratOG
A Your answer:
> 10–14 days post surgery
Correct answer:
> 10–14 days post surgery
The correct answer is 10–14 days a er surgery. Thermal injuries to
the ureter may result in delayed necrosis and/or fistula forma on
that will typically present clinically between 10 and 14 days
postopera vely. See Minas V, Gul N, Aust T, Doyle M, Rowlands D.
Urinary tract injuries in laparoscopic gynaecological surgery;
preven on, recogni on and management. The Obstetrician &
Gynaecologist 2014;16:19–28
(h p://onlinelibrary.wiley.com/doi/10.1111/tog.12073/abstract).
A Your answer:
> 1/10 to 1/100
Correct answer:
https://stratog.rcog.org.uk/sbas-part-2-mrcog-online-resource/gynaecology-sbas/gynaecology-page-2 5/8
11/21/2018 Gynaecology page 2 | StratOG
A Your answer:
> 6 cases per 1000 women
Correct answer:
> 6 cases per 1000 women
The correct answer is 6 addi onal cases per 1000 women.
Combined (estrogen and progesterone) HRT is associated with a
higher risk of breast cancer than estrogen-only HRT or bolone.
There are some discrepancies between the Million Women Study
(MWS) and Women's Health Ini a ve (WHI) study. Many of the
discrepencies can be explained by the popula ons studies. The WHI
study group was 16 000 women aged 50-79, 45% of whom had a
BMI of 30 or more. The MWS looked at 1 084 110 women aged 50-
64, only 18% of whom had a BMI of 30 or more See the Bri sh
Na onal Formulary. Hormone replacement therapy. Accessed online
February 2016
(h ps://www.medicinescomplete.com/mc/bnf/current/PHP78400-
sex-hormones.htm?
q=hormone%20replacement%20therapy&t=search&ss=text&p=1#_
hit).
https://stratog.rcog.org.uk/sbas-part-2-mrcog-online-resource/gynaecology-sbas/gynaecology-page-2 6/8
11/21/2018 Gynaecology page 2 | StratOG
A Your answer:
> 10%
Correct answer:
> 10%
The correct answer is 10%. BRCA1 and BRCA2 are highly penetrant
genes that account for 95% of families with both breast and ovarian
cancer. The cumula ve risk of ovarian cancer is lower in women
with BRCA2 at 11%, compared with BRCA1 where the risk is 39%.
See Devlin LA, Morrison PJ. Inherited gynaecological cancer
syndromes. The Obstetrician & Gynaecologist 2008;10:9–15
(h p://onlinelibrary.wiley.com/doi/10.1576/toag.10.1.009.27371/a
bstract) and Antoniou A, Pharoah PD, Narod S, Risch HA, Ey ord JE,
Hopper JL et al. Average risks of breast and ovarian cancer
associated with BRCA1 or BRCA2 muta ons detected in case Series
unselected for family history: a combined analysis of 22 studies. Am
J Hum Genet 2003;72:1117–30
(h p://www.ncbi.nlm.nih.gov/pmc/ar cles/PMC1180265/).
https://stratog.rcog.org.uk/sbas-part-2-mrcog-online-resource/gynaecology-sbas/gynaecology-page-2 7/8
11/21/2018 Gynaecology page 2 | StratOG
https://stratog.rcog.org.uk/sbas-part-2-mrcog-online-resource/gynaecology-sbas/gynaecology-page-2 8/8
11/21/2018 Gynaecology page 3 | StratOG
(/)
Gynaecology page 3
Assessment Total A empts: 1 Highest Score: 10 %
History
View last results (/quiz/latest/305536/299348) View highest
result (/quiz/highest/305536/299348) Retake quiz
(/quiz/retake/299348)
https://stratog.rcog.org.uk/sbas-part-2-mrcog-online-resource/gynaecology-sbas/gynaecology-page-3 1/7
11/21/2018 Gynaecology page 3 | StratOG
You have informed a 45-year-old that she has stage 3c ovarian cancer. She is keen
Q to know about her prognosis. What is the 5-year survival rate in UK for ovarian
cancer?
20–25%
30–35%
40–45%
50–55%
60–65%
A 16-year-old girl a ends the gynaecology clinic for heavy periods and confides
Q that she is being forced to undergo female genital mu la on (FGM) by her
parents. What is the es mated number of children at risk of FGM in the UK?
500
5000
10 000
20 000
50 000
A 40-year-old woman presents with severe pelvic pain. She has had a
Q myomectomy in the past through a ver cal abdominal incision to the level of the
umbilicus. To inves gate her pelvic pain, she undergoes a diagnos c laparoscopy
using the Palmer point of entry. Where is Palmer’s point?
https://stratog.rcog.org.uk/sbas-part-2-mrcog-online-resource/gynaecology-sbas/gynaecology-page-3 2/7
11/21/2018 Gynaecology page 3 | StratOG
https://stratog.rcog.org.uk/sbas-part-2-mrcog-online-resource/gynaecology-sbas/gynaecology-page-3 3/7
11/21/2018 Gynaecology page 3 | StratOG
Anterior colporrhaphy
Laparoscopic colposuspension
Levonelle®
Ulipristal acetate
https://stratog.rcog.org.uk/sbas-part-2-mrcog-online-resource/gynaecology-sbas/gynaecology-page-3 4/7
11/21/2018 Gynaecology page 3 | StratOG
Me ormin
Progesterone
A 17-year-old girl presents with a 12 hour history of lower abdominal pain. She
Q had unprotected intercourse a week ago, which was 6 days a er her last period.
Her pulse is 110 beats per minute, her blood pressure is 110/70 mmHg, her
temperature 37.8°C and she is tender over her lower abdomen, especially in the
right iliac fossa where there is rebound tenderness. There is cervical excita on. Her
Hb is 137g/l (normal 115–165) and her white cell count 17.6 x 10*9/l (normal 4–
11). What is the most likely diagnosis?
Acute appendici s
Ectopic pregnancy
Pelvic endometriosis
https://stratog.rcog.org.uk/sbas-part-2-mrcog-online-resource/gynaecology-sbas/gynaecology-page-3 5/7
11/21/2018 Gynaecology page 3 | StratOG
Right oophorectomy
This is the last ques on of this quiz. Press finish to submit your choice(s) and reveal the answer(s).
https://stratog.rcog.org.uk/sbas-part-2-mrcog-online-resource/gynaecology-sbas/gynaecology-page-3 6/7
11/21/2018 Gynaecology page 3 | StratOG
https://stratog.rcog.org.uk/sbas-part-2-mrcog-online-resource/gynaecology-sbas/gynaecology-page-3 7/7
11/21/2018 Gynaecology page 3 | StratOG
(/)
Gynaecology page 3
Assessment Total A empts: 2 Highest Score: 100 %
History
View last results (/quiz/latest/305536/299348) View highest
result (/quiz/highest/305536/299348) Retake quiz
(/quiz/retake/299348)
A Your answer:
> Full blood count (FBC)
Correct answer:
> Full blood count (FBC)
The correct answer is a full blood count (FBC). All women presen ng
with heavy menstrual bleeding should have FBC performed. An
ultrasound scan is not indicated unless the uterus is palpable
https://stratog.rcog.org.uk/sbas-part-2-mrcog-online-resource/gynaecology-sbas/gynaecology-page-3 1/8
11/21/2018 Gynaecology page 3 | StratOG
You have informed a 45-year-old that she has stage 3c ovarian cancer. She is
Q keen to know about her prognosis. What is the 5-year survival rate in UK for
ovarian cancer?
A Your answer:
> 40–45%
Correct answer:
> 40–45%
The correct answer is 40–45%. As with the majority of cancers,
rela ve survival for ovarian cancer is improving. Much of the
increase occurred during the 1980s and 1990s, and appears to be
leveling off in the 2000s. The significant increase in 1-year survival is
likely to be the result of greater use of pla num-based
chemotherapy. One-year rela ve survival rates for ovarian cancer
increased from 42% in England and Wales in 1971–1975 to 72.3% in
England in 2005–2009. The 5-year survival rate for advanced ovarian
cancer in 2005–2009 was 43%. See Cancer Research UK. Ovarian
cancer survival sta s cs. Accessed online July 2015
(h p://www.cancerresearchuk.org/cancer-
info/cancerstats/types/ovary/survival/#trends).
A 16-year-old girl a ends the gynaecology clinic for heavy periods and
Q confides that she is being forced to undergo female genital mu la on
(FGM) by her parents. What is the es mated number of children at risk of
FGM in the UK?
A Your answer:
> 20 000
https://stratog.rcog.org.uk/sbas-part-2-mrcog-online-resource/gynaecology-sbas/gynaecology-page-3 2/8
11/21/2018 Gynaecology page 3 | StratOG
Correct answer:
> 20 000
The correct answer is 20 000. It is es mated that 20 000 girls in the
UK are at risk of FGM, usually through travelling abroad to facilitate
the procedure. It is important that the safeguarding team are
informed when a woman who has undergone FGM themselves
delivers a female child. See Royal College of Obstetricians and
Gynaecologists. Female genital mu la on and its management.
Green-top Guideline 53. London: RCOG; 2009
(h ps://www.rcog.org.uk/en/guidelines-research-
services/guidelines/gtg53/).
A 40-year-old woman presents with severe pelvic pain. She has had a
Q myomectomy in the past through a ver cal abdominal incision to the level
of the umbilicus. To inves gate her pelvic pain, she undergoes a diagnos c
laparoscopy using the Palmer point of entry. Where is Palmer’s point?
A Your answer:
> 3 cm below the le costal margin in the midclavicular line
Correct answer:
> 3 cm below the le costal margin in the midclavicular line
The correct answer is 3 cm below the le costal margin in the
midclavicular line. Palmer’s point should be used if there is a high
suspicion of adhesions. Adhesions are found in up to 50% of women
following midline laparotomy but are rarely found in the le upper
quadrant. The usual trocar and cannulae can be inserted under
direct vision or following dissec on of any adhesions seen. If there
are two failed a empts at insuffla on then u lising Palmer’s point
or the open Hasson technique should be used. See Royal College of
Obstetricians and Gynaecologists. Preven ng entry related
gynaecological laparoscopic injuries. Green-top Guideline 49.
London: RCOG; 2008. (h ps://www.rcog.org.uk/en/guidelines-
research-services/guidelines/gtg49/)
https://stratog.rcog.org.uk/sbas-part-2-mrcog-online-resource/gynaecology-sbas/gynaecology-page-3 3/8
11/21/2018 Gynaecology page 3 | StratOG
A Your answer:
> Counsel her regarding expectant management of miscarriage
Correct answer:
> Counsel her regarding expectant management of miscarriage
The correct answer is counsel her regarding expectant management
of miscarriage. Expectant management should be offered as first
line management for all women with a confirmed diagnosis of
miscarriage, taking into account if she is at increased risk of
haemorrhage (e.g. late first trimester), has risks associated with
haemorrhage (e.g. unable to have a blood transfusion), evidence of
infec on, or her personal wishes. Mifepristone is not indicated in
management of a non viable pregnancy. See Na onal Ins tute for
Health and Clinical Excellence. Ectopic pregnancy and miscarriage.
CG154. NICE; 2012 (h p://www.nice.org.uk/guidance/CG154).
A Your answer:
> Synthe c mid-urethral tape
Correct answer:
> Synthe c mid-urethral tape
https://stratog.rcog.org.uk/sbas-part-2-mrcog-online-resource/gynaecology-sbas/gynaecology-page-3 4/8
11/21/2018 Gynaecology page 3 | StratOG
A Your answer:
> A copper bearing intrauterine device
Correct answer:
> A copper bearing intrauterine device
The correct answer is a copper bearing intrauterine device. The
choice of EC depends on the length of me since unprotected sexual
intercourse. All forms are not effec ve a er 6 days except for the
copper-bearing intrauterine device, and only in the circumstance
that it is within 5 days of the earliest es mated date of ovula on.
See Faculty of Sexual and Reproduc ve Healthcare. Emergency
contracep on. Clinical Guidance. London: FSRH; 2011
(h p://www.fsrh.org/pages/Clinical_Guidance_2.asp).
https://stratog.rcog.org.uk/sbas-part-2-mrcog-online-resource/gynaecology-sbas/gynaecology-page-3 5/8
11/21/2018 Gynaecology page 3 | StratOG
A Your answer:
> Aspirin and heparin
Correct answer:
> Aspirin and heparin
The correct answer is aspirin and heparin. An phospholipid
syndrome is present in 15% of women with recurrent miscarriage.
Without treatment, the live birth rate has been reported to be as
low as 10%. Cor costeroids and intravenous immunoglobulin are
associated with significant maternal and fetal morbidity. Despite the
associa on between PCOS and miscarriage that is a ributed to
insulin resistance and hyperinsulinaemia, a meta-analysis of 117
randomised controlled trials showed no reduc on in the rate of
miscarriage in those women prescribed me ormin. See Royal
College of Obstetricians and Gynaecologists. Inves ga on and
treatment of couples with recurrent miscarriage. Green-top
Guideline 17. London: RCOG; 2011
(h ps://www.rcog.org.uk/en/guidelines-research-
services/guidelines/gtg17/).
A Your answer:
> Acute appendici s
Correct answer:
> Acute appendici s
The correct answer is acute appendici s. The white count and mild
pyrexia suggest an infec on and the localisa on to the right iliac
fossa makes this more likely to be appendici s.
https://stratog.rcog.org.uk/sbas-part-2-mrcog-online-resource/gynaecology-sbas/gynaecology-page-3 6/8
11/21/2018 Gynaecology page 3 | StratOG
A Your answer:
> Repeat scan and Ca 125 test in 4 months
Correct answer:
> Repeat scan and Ca 125 test in 4 months
The correct answer is repeat scan and Ca 125 test in 4 months. The
risk of malignancy index (RMI) is zero since the cyst is simple and it
measures less than 5 cm. Therefore, monitoring for 12 months is all
that is required. See Royal College of Obstetricians and
Gynaecologists. Ovarian cysts in postmenopausal women. Green-top
Guideline 34. London: RCOG; 2003
(h ps://www.rcog.org.uk/en/guidelines-research-
services/guidelines/gtg34/).
https://stratog.rcog.org.uk/sbas-part-2-mrcog-online-resource/gynaecology-sbas/gynaecology-page-3 7/8
11/21/2018 Gynaecology page 3 | StratOG
https://stratog.rcog.org.uk/sbas-part-2-mrcog-online-resource/gynaecology-sbas/gynaecology-page-3 8/8
11/21/2018 Gynaecology page 4 | StratOG
(/)
Gynaecology page 4
Assessment Total A empts: 2 Highest Score: 90 %
History
View last results (/quiz/latest/305490/299349) View highest
result (/quiz/highest/305490/299349) Retake quiz
(/quiz/retake/299349)
A 67-year-old woman is referred to the rapid access clinic with a 2 day history of
Q postmenopausal bleeding, which has since resolved. She is otherwise fit and well.
The endometrial thickness is 7 mm on transvaginal ultrasound scan, the
endometrium appears polypoidal at hysteroscopy and histology on an endometrial
sample is reported as showing irregular and ghtly packed glands with large and
vesicular nuclei containing prominent nucleoli. What is the most appropriate
management for this woman?
Bilateral oophorectomy
Combined estrogen and progestogen hormone replacement therapy
Expectant management
Hysterectomy
Inser on of a levonorgestrel-releasing intrauterine system
https://stratog.rcog.org.uk/sbas-part-2-mrcog-online-resource/gynaecology-sbas/gynaecology-page-4 1/7
11/21/2018 Gynaecology page 4 | StratOG
A 40-year-old woman has regular heavy menstrual bleeding. The history and
Q inves ga ons indicate that pharmacological treatment is appropriate. Her GP has
tried tranexamic acid without success. What is the most appropriate next
pharmaceu cal treatment?
Etamsylate
46 XY
46 YY
46 YYY
69 XYY
69 YYY
https://stratog.rcog.org.uk/sbas-part-2-mrcog-online-resource/gynaecology-sbas/gynaecology-page-4 2/7
11/21/2018 Gynaecology page 4 | StratOG
You see a 38-year-old woman with a 2.5 cm malignant tumour on her cervix and
Q no extracervical disease on imaging. She is fit and healthy. What is her best
treatment op on?
Radical hysterectomy
Radical trachelectomy
Radiotherapy
1 day
3 days
5 days
7 days
14 days
https://stratog.rcog.org.uk/sbas-part-2-mrcog-online-resource/gynaecology-sbas/gynaecology-page-4 3/7
11/21/2018 Gynaecology page 4 | StratOG
Endometrial sampling
HPV tes ng
No change in her management
Referral to colposcopy
A 46-year-old nulliparous woman has been referred by her GP having been treated
Q for heavy regular menstrual bleeding with cyclical progestogens for a period of 6
months. The treatment has failed to improve her symptoms. What is the most
appropriate next line of management?
Endometrial biopsy
Levonorgestrel intrauterine system
Pelvic ultrasound
Tranexamic acid
https://stratog.rcog.org.uk/sbas-part-2-mrcog-online-resource/gynaecology-sbas/gynaecology-page-4 4/7
11/21/2018 Gynaecology page 4 | StratOG
Copper IUCD
Mirena IUS
Levonorgestrel (LNG)
Mifepristone
Ulipristal acetate (UA)
One-tenth of cases
One-fi h of cases
One-quarter of cases
One-third of cases
One-half of cases
https://stratog.rcog.org.uk/sbas-part-2-mrcog-online-resource/gynaecology-sbas/gynaecology-page-4 5/7
11/21/2018 Gynaecology page 4 | StratOG
4%
8%
12%
16%
20%
This is the last ques on of this quiz. Press finish to submit your choice(s) and reveal the answer(s).
https://stratog.rcog.org.uk/sbas-part-2-mrcog-online-resource/gynaecology-sbas/gynaecology-page-4 6/7
11/21/2018 Gynaecology page 4 | StratOG
https://stratog.rcog.org.uk/sbas-part-2-mrcog-online-resource/gynaecology-sbas/gynaecology-page-4 7/7
11/21/2018 Gynaecology page 4 | StratOG
(/)
Gynaecology page 4
Assessment Total A empts: 5 Highest Score: 100 %
History
View last results (/quiz/latest/305490/299349) View highest
result (/quiz/highest/305490/299349) Retake quiz
(/quiz/retake/299349)
A Your answer:
> Hysterectomy
Correct answer:
> Hysterectomy
https://stratog.rcog.org.uk/sbas-part-2-mrcog-online-resource/gynaecology-sbas/gynaecology-page-4 1/8
11/21/2018 Gynaecology page 4 | StratOG
A Your answer:
> Levonorgestrel-releasing intrauterine system (LNG-IUS)
Correct answer:
> Levonorgestrel-releasing intrauterine system (LNG-IUS)
The correct answer is levonorgestrel-releasing intrauterine system
(LNG-IUS). The LNG-IUS is first line treatment in women complaining
of heavy menstrual bleeding and NICE recommends it's use before
tranexamic acid. See Na onal Ins tute for Health and Clinical
Excellence. Heavy menstrual bleeding. CG44. NICE; 2007
(h p://www.nice.org.uk/guidance/CG44).
https://stratog.rcog.org.uk/sbas-part-2-mrcog-online-resource/gynaecology-sbas/gynaecology-page-4 2/8
11/21/2018 Gynaecology page 4 | StratOG
A Your answer:
> 69 XYY
Correct answer:
> 69 XYY
The correct answer is 69 XYY. Complete moles are usually diploid
and all chromosomes are of paternal origin. Par al molar
pregnancies are usually triploid, with the addi onal set of
chromosomes of maternal origin. Incidence varies worldwide,
ranging from 2 in 1000 pregnancies in Japan to 0.6–1.1 per 1000 in
Europe and North America. See Royal College of Obstetricians and
Gynaecologists. Gesta onal trophoblas c disease. Green-top
Guideline 38. London: RCOG; 2010
(h ps://www.rcog.org.uk/en/guidelines-research-
services/guidelines/gtg38/).
You see a 38-year-old woman with a 2.5 cm malignant tumour on her cervix
Q and no extracervical disease on imaging. She is fit and healthy. What is her
best treatment op on?
A Your answer:
> Radical hysterectomy and bilateral pelvic
lymphadenectomy
Correct answer:
> Radical hysterectomy and bilateral pelvic
lymphadenectomy
The correct answer is radical hysterectomy and bilateral pelvic
lymphadenectomy. Radical surgery is recommended in stage 1B1
disease if there is no contraindica on to surgery. Radical
trachelectomy can only be offered for fer lity sparing in tumours
https://stratog.rcog.org.uk/sbas-part-2-mrcog-online-resource/gynaecology-sbas/gynaecology-page-4 3/8
11/21/2018 Gynaecology page 4 | StratOG
A Your answer:
> 7 days
Correct answer:
> 7 days
The correct answer is 7 days. You should assess the risks and
benefits of stopping pre-exis ng an platelet therapy 1 week before
surgery. Consider involving the mul disciplinary team in the
assessment. See: Na onal Ins tute for Health and Clinical
Excellence. Venous thromboembolism: reducing the risk. CG92.
London: NICE; 2010 (h p://www.nice.org.uk/guidance/CG92).
A Your answer:
> Referral to colposcopy
Correct answer:
> Referral to colposcopy
https://stratog.rcog.org.uk/sbas-part-2-mrcog-online-resource/gynaecology-sbas/gynaecology-page-4 4/8
11/21/2018 Gynaecology page 4 | StratOG
A Your answer:
> Endometrial biopsy
Correct answer:
> Endometrial biopsy
The correct answer is endometrial biopsy. Endometrial biopsy
should be performed if a women over 45 years of age fails to
respond to first line treatment. See: Na onal Ins tute for Health
and Clinical Excellence. Heavy menstrual bleeding. CG44. London:
NICE; 2007 (h p://www.nice.org.uk/guidance/CG44).
A Your answer:
> Ulipristal acetate (UA)
https://stratog.rcog.org.uk/sbas-part-2-mrcog-online-resource/gynaecology-sbas/gynaecology-page-4 5/8
11/21/2018 Gynaecology page 4 | StratOG
Correct answer:
> Ulipristal acetate (UA)
The correct asnwer is ulipristal acetate (UA). The Mirena coil is not
licensed for EC. LNG is recommended only within 72 hours of UPSI.
A copper IUCD can be used within 5 days of first UPSI in a cycle but
is not indicated in the presence of a uterine anomaly. Mifepristone
is not licensed for EC in the UK. UA is licensed for use within 120
hours of UPSI so is the recommended choice. See Faculty of Sexual
and Reproduc ve Health Care. Emergency contracep on. London:
FSRH; 2011
(h p://www.fsrh.org/pdfs/CEUguidanceEmergencyContracep on11
.pdf).
A Your answer:
> One-third of cases
Correct answer:
> One-third of cases
The correct answer is one-third of cases. Approximately one-third of
cases reported as borderline tumours on frozen sec on are later
reclassified as invasive tumours. For the older women with no
fer lity concerns, if frozen sec on is reported as a borderline
tumour then complete staging should be undertaken. See Bagade P,
Edmondson R, Nayar A. Management of borderline ovarian
tumours. The Obstetrician & Gynaecologist 2012;14:115–20
(h p://onlinelibrary.wiley.com/doi/10.1111/j.1744-
4667.2012.00102.x/abstract).
https://stratog.rcog.org.uk/sbas-part-2-mrcog-online-resource/gynaecology-sbas/gynaecology-page-4 6/8
11/21/2018 Gynaecology page 4 | StratOG
A Your answer:
> 4%
Correct answer:
> 4%
The correct answer is 4%. It is important to be able to counsel
pa ents appropriately regarding their risk of malignancy and not to
confuse complex hyperplasia with complex atypical hyperplasia. See
Palmer JE, Perunovic B, Tidy JA. Endometrial hyperplasia. The
Obstetrician & Gynaecologist 2008;10:211–6
(h p://onlinelibrary.wiley.com/doi/10.1576/toag.10.4.211.27436/a
bstract).
https://stratog.rcog.org.uk/sbas-part-2-mrcog-online-resource/gynaecology-sbas/gynaecology-page-4 7/8
11/21/2018 Gynaecology page 4 | StratOG
https://stratog.rcog.org.uk/sbas-part-2-mrcog-online-resource/gynaecology-sbas/gynaecology-page-4 8/8
11/21/2018 Gynaecology page 5 | StratOG
(/)
Gynaecology page 5
Assessment Total A empts: 2 Highest Score: 100 %
History
View last results (/quiz/latest/305554/299350) View highest
result (/quiz/highest/305554/299350) Retake quiz
(/quiz/retake/299350)
A 36-year-old parous woman was diagnosed with stage 3 endometriosis. She was
Q on GnRH (gonadotrophin releasing hormone) analogue for 12 months.
Subsequently she had laparoscopic excision of recto-vaginal endometriosis. She
con nues to be in pain despite medical and surgical management. What is the
next most appropriate management op on for her?
Aromatase inhibitors
Danazol
https://stratog.rcog.org.uk/sbas-part-2-mrcog-online-resource/gynaecology-sbas/gynaecology-page-5 1/6
11/21/2018 Gynaecology page 5 | StratOG
Citalopram
Metaprolol
Nifedipine
Phentolamine
Venlafaxine
A 26-year-old woman has been admi ed with late onset severe ovarian
Q hypers mula on syndrome (OHSS) 10 days a er embryo transfer in an IVF cycle.
She reports generalised abdominal pain and sickness for 2 days. Abdominal
examina on revealed significant ascites, whilst abdominal ultrasound showed
bilateral enlarged ovaries with a maximal diameter of 10 cm. Which of the
following combina on of blood results is commonly observed on admission?
https://stratog.rcog.org.uk/sbas-part-2-mrcog-online-resource/gynaecology-sbas/gynaecology-page-5 2/6
11/21/2018 Gynaecology page 5 | StratOG
Mito c figures
Peritoneal implants
Renogram
A 24-year-old woman in her first pregnancy a ends the antenatal clinic. Her
Q community midwife has referred her to a Consultant clinic as she disclosed having
had female genital mu la on (FGM) at 8 years of age. Which one of the following
countries is this woman LEAST likely to originate from?
https://stratog.rcog.org.uk/sbas-part-2-mrcog-online-resource/gynaecology-sbas/gynaecology-page-5 3/6
11/21/2018 Gynaecology page 5 | StratOG
Egypt
Eritrea
Nigeria
Somalia
Sudan
Aromatase inhibitor
Progestogen antagonist
Prostaglandin
Inform the GP
Escherichia coli
Haemophilus influenzae
Staphylococcal aureus
Streptococcus milleri
Convert to laparotomy
https://stratog.rcog.org.uk/sbas-part-2-mrcog-online-resource/gynaecology-sbas/gynaecology-page-5 5/6
11/21/2018 Gynaecology page 5 | StratOG
This is the last ques on of this quiz. Press finish to submit your choice(s) and reveal the answer(s).
https://stratog.rcog.org.uk/sbas-part-2-mrcog-online-resource/gynaecology-sbas/gynaecology-page-5 6/6
11/21/2018 Gynaecology page 5 | StratOG
(/)
Gynaecology page 5
Assessment Total A empts: 2 Highest Score: 100 %
History
View last results (/quiz/latest/305554/299350) View highest
result (/quiz/highest/305554/299350) Retake quiz
(/quiz/retake/299350)
A Your answer:
> Aromatase inhibitors
Correct answer:
> Aromatase inhibitors
https://stratog.rcog.org.uk/sbas-part-2-mrcog-online-resource/gynaecology-sbas/gynaecology-page-5 1/8
11/21/2018 Gynaecology page 5 | StratOG
A Your answer:
> Venlafaxine
Correct answer:
> Venlafaxine
The correct answer is Venlafaxine. Selec ve serotonin and
noradrenaline reuptake inhibitors are the drugs used most
commonly to alleviate vasomotor symptoms. The most convincing
data relates to venlafaxine, although this was a short study. See
Royal College of Obstetricians and Gynaecologists. Alterna ves to
HRT for the management of symptoms of the menopause. Scien fic
Impact Paper 6. London: RCOG; 2010
(h ps://www.rcog.org.uk/en/guidelines-research-
services/guidelines/sip6/).
A 26-year-old woman has been admi ed with late onset severe ovarian
Q hypers mula on syndrome (OHSS) 10 days a er embryo transfer in an IVF
cycle. She reports generalised abdominal pain and sickness for 2 days.
Abdominal examina on revealed significant ascites, whilst abdominal
https://stratog.rcog.org.uk/sbas-part-2-mrcog-online-resource/gynaecology-sbas/gynaecology-page-5 2/8
11/21/2018 Gynaecology page 5 | StratOG
A Your answer:
> Haematocrit increased, fibrinogen increased, albumin decreased
Correct answer:
> Haematocrit increased, fibrinogen increased, albumin decreased
The correct answer is haematocrit increased, fibrinogen increased,
albumin decreased. Severe OHSS is usually associated with an
increased capillary permeability resul ng in a reduc on of
intravascular volume and haemoconcentra on (increase
haematocrit), and a shi of fluid into the third compartment (a
reduc on of serum albumin concentra ons). The woman is at risk of
developing thrombosis (increase fibrinogen levels). See Prakash A,
Mathur R. Ovarian hypers mula on syndrome. The Obstetrician &
Gynaecologist 2013;15:31–5
(h p://onlinelibrary.wiley.com/doi/10.1111/j.1744-
4667.2012.00153.x/abstract).
A Your answer:
> Absence of stromal invasion
Correct answer:
> Absence of stromal invasion
The correct asnwer is absence of stromal invasion. Borderline
tumours are o en found following primary surgery in younger
women. They show higher prolifera ve ac vity than benign
tumours, but do not show stromal invasion. They cons tute 10–15%
of ovarian neoplasms. Serous borderline tumours are the most
common and are o en (30%) bilateral. See Bagade P, Edmondson R,
Nayar A. Management of borderline ovarian tumours. The
https://stratog.rcog.org.uk/sbas-part-2-mrcog-online-resource/gynaecology-sbas/gynaecology-page-5 3/8
11/21/2018 Gynaecology page 5 | StratOG
A Your answer:
> Computerised tomography intravenous urogram
Correct answer:
> Computerised tomography intravenous urogram
The correct answer is computerised tomography intravenous
urogram. Endometriosis increases the risk of injury to the urinary
tract. An acute injury usually presents within 48 hours with diffuse
abdominal pain, distension and ileus. The chemical peritoni s has
more subtle symptoms compared with peritoni s secondary to
faeces or infec on. A CT scan with contrast will usually demonstrate
a uroperitoneum and may show direct evidence of the injury. MRI is
useful in late presenta ons where a fistula is suspected. See Minas
V, Gul N, Aust T, Doyle M, Rowlands D. Urinary tract injuries in
laparoscopic gynaecological surgery; preven on, recogni on and
management. The Obstetrician & Gynaecologist 2014;16:19–28
(h p://onlinelibrary.wiley.com/doi/10.1111/tog.12073/full).
A 24-year-old woman in her first pregnancy a ends the antenatal clinic. Her
Q community midwife has referred her to a Consultant clinic as she disclosed
having had female genital mu la on (FGM) at 8 years of age. Which one of
the following countries is this woman LEAST likely to originate from?
A Your answer:
> Nigeria
https://stratog.rcog.org.uk/sbas-part-2-mrcog-online-resource/gynaecology-sbas/gynaecology-page-5 4/8
11/21/2018 Gynaecology page 5 | StratOG
Correct answer:
> Nigeria
The correct answer is Nigeria. The prevalence of FGM varies by
country. The type of FGM also varies and the more severe types are
commonest in Somalia. Somalia has the highest incidence at 98–
100% of girls and this is usually type III. Royal College of
Obstetricians and Gynaecologists. Female genital mu la on and its
management. Green-top Guideline 53. London: RCOG; 2009
(h ps://www.rcog.org.uk/en/guidelines-research-
services/guidelines/gtg53/).
A Your answer:
> Progestogen antagonist
Correct answer:
> Progestogen antagonist
The correct answer is progestogen antagonist. Ulipristal acetate has
been used as a drug for emergency contracep on. It has recently
been licensed for use in reducing the size of fibroids prior to surgery
and it does this by inducing apoptosis in the cells. See the Bri sh
Na onal Formulary
(h ps://www.medicinescomplete.com/about/)for more details.
A Your answer:
> Encourage her to inform her parents
https://stratog.rcog.org.uk/sbas-part-2-mrcog-online-resource/gynaecology-sbas/gynaecology-page-5 5/8
11/21/2018 Gynaecology page 5 | StratOG
Correct answer:
> Encourage her to inform her parents
The correct answer is encourage her to inform her parents. Fraser
guidelines relate to a case in 1984 – Gillick v West Norfolk – and
provide a framework for dealing with children under the age of 16.
It revolves around whether a child is capable of making a reasonable
assessment of the advantages and disadvantages of treatment and
thus their ability to consent to treatment. In his guidance Fraser
stated that a doctor could prescribe contracep ves "provided he is
sa sfied in the following criteria:
1. That the girl (although under the age of 16 years of age) will
understand his advice
2. That he cannot persuade her to inform her parents or to allow
him to inform the parents that she is seeking contracep ve
advice
3. That she is very likely to con nue having sexual intercourse with
or without contracep ve treatment
4. That unless she receives contracep ve advice or treatment her
physical or mental health, or both, are likely to suffer
5. That her best interests require him to give her contracep ve
advice, treatment, or both, without the parental consent."
The same guidelines relate to termina on of pregnancy. See Royal
College of Obstetricians and Gynaecologists. The care of women
reques ng induced abor on. Evidence-based Clinical Guideline 7.
London: RCOG; 2011 (h ps://www.rcog.org.uk/en/guidelines-
research-services/guidelines/the-care-of-women-reques ng-
induced-abor on/).
A Your answer:
> Staphylococcal aureus
Correct answer:
> Staphylococcal aureus
https://stratog.rcog.org.uk/sbas-part-2-mrcog-online-resource/gynaecology-sbas/gynaecology-page-5 6/8
11/21/2018 Gynaecology page 5 | StratOG
A Your answer:
> Visualise the primary trocar site from a secondary port site
Correct answer:
> Visualise the primary trocar site from a secondary port site
The correct answer is visualise the primary trocar site from a
secondary port site. If there are adhesions within the abdomen it is
advisable to check the umbilical port by inspec ng it through a
preferably 5 mm scope via a secondary port. If damage has occurred
seek surgical advice. See Royal College of Obstetricians and
Gynaecologists. Preven ng entry-related gynaecological
laparoscopic injuries. Green-top Guideline 49. London: RCOG; 2008.
(h ps://www.rcog.org.uk/en/guidelines-research-
services/guidelines/gtg49/)
https://stratog.rcog.org.uk/sbas-part-2-mrcog-online-resource/gynaecology-sbas/gynaecology-page-5 7/8
11/21/2018 Gynaecology page 5 | StratOG
https://stratog.rcog.org.uk/sbas-part-2-mrcog-online-resource/gynaecology-sbas/gynaecology-page-5 8/8
11/21/2018 Gynaecology page 6 | StratOG
(/)
Gynaecology page 6
Assessment Total A empts: 2 Highest Score: 100 %
History
View last results (/quiz/latest/306244/301527) View highest
result (/quiz/highest/306244/301527) Retake quiz
(/quiz/retake/301527)
Add extra progesterone cover for 5 days per month during the pill-free
interval
Advise that this is normal and review in a further 3 months
Change to a COCP with 30 micrograms of ethinyl estradiol and reassess
a er 3 months
https://stratog.rcog.org.uk/sbas-part-2-mrcog-online-resource/gynaecology-sbas/gynaecology-page-6 1/7
11/21/2018 Gynaecology page 6 | StratOG
Stop the pill and monitor her symptoms before trying alterna ve hormonal
contracep on
AFS score
A 32-year-old woman had normal vaginal delivery 6 months ago. She complains of
Q stress urinary incon nence on coughing and sneezing. Abdominal and pelvic
examina ons were unremarkable and stress incon nence was demonstrable.
What is the most appropriate strategy to manage her stress incon nence?
https://stratog.rcog.org.uk/sbas-part-2-mrcog-online-resource/gynaecology-sbas/gynaecology-page-6 2/7
11/21/2018 Gynaecology page 6 | StratOG
Ring pessary
Urodynamics
Computed tomography
Transvaginal ultrasound
A 30-year-old woman was diagnosed with polycys c ovary syndrome presents with
Q primary subfer lity of 4 years. Her BMI is 20. Her partner’s semen analysis is
sa sfactory (WHO criteria 2010). Hysterosalpingography has confirmed bilateral
tubal patency. What is the most appropriate first-line treatment for this couple?
https://stratog.rcog.org.uk/sbas-part-2-mrcog-online-resource/gynaecology-sbas/gynaecology-page-6 3/7
11/21/2018 Gynaecology page 6 | StratOG
The most common site of bladder injury is in the midline above the
interureteric bar
Thermal injuries usually present within 72 hours with uroperitoneum or
vesico–vaginal fistula
https://stratog.rcog.org.uk/sbas-part-2-mrcog-online-resource/gynaecology-sbas/gynaecology-page-6 4/7
11/21/2018 Gynaecology page 6 | StratOG
10–15%
20–25%
30–35%
40–45%
50–55%
https://stratog.rcog.org.uk/sbas-part-2-mrcog-online-resource/gynaecology-sbas/gynaecology-page-6 5/7
11/21/2018 Gynaecology page 6 | StratOG
0.6%
1.2%
2%
3.1%
4.3%
5–8%
9–12%
13–16%
17–20%
21–24%
This is the last ques on of this quiz. Press finish to submit your choice(s) and reveal the answer(s).
https://stratog.rcog.org.uk/sbas-part-2-mrcog-online-resource/gynaecology-sbas/gynaecology-page-6 6/7
11/21/2018 Gynaecology page 6 | StratOG
© 2018
https://stratog.rcog.org.uk/sbas-part-2-mrcog-online-resource/gynaecology-sbas/gynaecology-page-6 7/7
11/21/2018 Gynaecology page 6 | StratOG
(/)
Gynaecology page 6
Assessment Total A empts: 2 Highest Score: 100 %
History
View last results (/quiz/latest/306244/301527) View highest
result (/quiz/highest/306244/301527) Retake quiz
(/quiz/retake/301527)
A Your answer:
> Advise that this is normal and review in a further 3 months
Correct answer:
> Advise that this is normal and review in a further 3 months
https://stratog.rcog.org.uk/sbas-part-2-mrcog-online-resource/gynaecology-sbas/gynaecology-page-6 1/8
11/21/2018 Gynaecology page 6 | StratOG
A Your answer:
> Pelvic Organ Prolapse Quan fica on System (POP–Q)
Correct answer:
> Pelvic Organ Prolapse Quan fica on System (POP–Q)
The answer is POP-Q.
A Your answer:
> Pelvic floor muscle training
Correct answer:
> Pelvic floor muscle training
The answer is pelvic floor muscle training. You should offer a trial of
supervised pelvic floor muscle training of at least 3 months'
dura on as first-line treatment to women with stress or mixed
urinary incon nence.
A Your answer:
> Transvaginal ultrasound
Correct answer:
> Transvaginal ultrasound
The answer is transvaginal ultrasound. Ultrasound is a well-
established test to assess postmenopausal cysts, achieving a
sensi vity of 89% and a specificity of 73%.
https://stratog.rcog.org.uk/sbas-part-2-mrcog-online-resource/gynaecology-sbas/gynaecology-page-6 3/8
11/21/2018 Gynaecology page 6 | StratOG
A Your answer:
> Ovula on induc on with clomifene citrate
Correct answer:
> Ovula on induc on with clomifene citrate
The answer is ovula on induc on with clomifene citrate.
Na onal Ins tute for Health and Care Excellence. Fer lity
Problems: Assessment and Treatment. CG156. NICE; 2013.
(h ps://www.nice.org.uk/guidance/cg156)
Gorthi S, Balen A, Tang T. Current issues in ovula on
induc on. The Obstetrician & Gynaecologist 2012;14:188–96.
(h ps://stratog.rcog.org.uk/sites/default/files/Gorthi_et_al-
2012-The_Obstetrician_%26_Gynaecologist_5_0.pdf)
A Your answer:
> The most common site of bladder injury is in the midline above
the interureteric bar
Correct answer:
> The most common site of bladder injury is in the midline above
the interureteric bar
The answer is that the most common site of bladder injury is in the
midline above the interureteric bar.
https://stratog.rcog.org.uk/sbas-part-2-mrcog-online-resource/gynaecology-sbas/gynaecology-page-6 4/8
11/21/2018 Gynaecology page 6 | StratOG
A Your answer:
> 30–35%
Correct answer:
> 30–35%
The answer is 30–35%.
A Your answer:
> The Million Women Study
https://stratog.rcog.org.uk/sbas-part-2-mrcog-online-resource/gynaecology-sbas/gynaecology-page-6 5/8
11/21/2018 Gynaecology page 6 | StratOG
Correct answer:
> The Million Women Study
The answer is The Million Women Study. Women aged 50–64 years
in the UK a ending the NHS breast screening programme were
invited to take part in this study, and this was subsequently followed
by the comple on of a ques onnaire. A significant increased risk of
breast cancer was seen in the women on combined HRT (estrogen
and progestogens) compared with estrogen only and bolone.
A Your answer:
> 1.2%
Correct answer:
> 1.2%
The answer is 1.2%.
https://stratog.rcog.org.uk/sbas-part-2-mrcog-online-resource/gynaecology-sbas/gynaecology-page-6 6/8
11/21/2018 Gynaecology page 6 | StratOG
A Your answer:
> 13–16%
Correct answer:
> 13–16%
The answer is 13–16%. See Iewin DE, Milson I, Hunskaar S, Reilley K,
Kopp Z, Herschorn S et al. Popula on-based survey of urinary
incon nence, overac ve bladder, and other lower urinary tract
symptoms in five countries: results of EPIC study. Eur Urol
2006;50:1306–14 [Abstract]
(h p://www.sciencedirect.com/science/ar cle/pii/S0302283806011
16X).
https://stratog.rcog.org.uk/sbas-part-2-mrcog-online-resource/gynaecology-sbas/gynaecology-page-6 7/8
11/21/2018 Gynaecology page 6 | StratOG
https://stratog.rcog.org.uk/sbas-part-2-mrcog-online-resource/gynaecology-sbas/gynaecology-page-6 8/8